You are on page 1of 27

VEDANGA INSTITUTE FOR CIVIL SERVICES

COMPILATIONS
PRELIMS
2022 July
AS EXAM PREPARATION
www.insightsactivelearn.com | www

U-135, Ground Floor, 2nd Floor, Vikas Marg, Baba Complex,


Near Laxmi Nagar Metro Station Gate No. 3&4, Shakarpur, Delhi-92
info@vedanga.co.in
1. Consider the following statements: 1. िन िल खत कथनों पर िवचार कीिजएः
1. Constitution of India vests the superintendence, direction 1. भारत का सं िवधान रा सभा के महासिचव म भारत के उपरा पित
and control of the conduct of election to the office of the Vice- के कायालय के चुनाव के सं चालन का अधी ण, िनदशन और िनयं ण
President of India in the Secretary General of the Rajya
िनिहत करता है ।
Sabha.
2. भारत के सं िवधान के अनु े द 66 के अनुसार, उपरा पित का
2. As per Article 66 of the Constitution of India, the Vice-
President is elected by the members of the Electoral College. चुनाव िनवाचक मंडल के सद ों ारा िकया जाता है ।
Which of the statements given above is/are correct? ऊपर िदए गए कथनों म से कौन सा/से सही है /ह?
(a) 1 only (ए) केवल 1
(b) 2 only (बी) केवल 2
(c) Both 1 and 2 (सी) 1 और 2 दोनों
(d) Neither 1 nor 2
(डी) न तो 1 और न ही 2
Ans: (b)
उ र: (बी)
Explanation:
• Article 324 of the Constitution read with the Presidential ा ा:
and VicePresidential Elections Act, 1952 and the Presidential • रा पित और उपरा पित चुनाव अिधिनयम, 1952 और रा पित और
and Vice-Presidential Elections Rules, 1974, vests the उपरा पित चुनाव िनयम, 1974 के साथ पढ़े जाने वाले संिवधान के
superintendence, direction and control of the conduct of अनु े द 324 म भारत के उपरा पित के कायालय के चुनाव के
election to the office of the Vice-President of India in the सं चालन का अधी ण, िनदशन और िनयं ण िनिहत है । भारत के चुनाव
Election Commission of India.
आयोग म।
• As per Article 66 of the Constitution of India, the Vice-
• भारत के सं िवधान के अनु े द 66 के अनुसार, उपरा पित का चुनाव
President is elected by the members of the Electoral College.
• Electoral College consists of: िनवाचक मंडल के सद ों ारा िकया जाता है ।
o Elected members of Rajya Sabha. • िनवाचक मंडल म शािमल ह:
o Nominated members of Rajya Sabha. 0 रा सभा के िनवािचत सद ।
o Elected members of Lok Sabha. 0 रा सभा के मनोनीत सद ।
0 लोकसभा के िनवािचत सद ।
2. Consider the following statements
1. Vice-President is a member and chairman of Council of
2. िन िल खत कथनों पर िवचार कर
States.
1. उपरा पित रा प रषद का सद और अ होता है ।
2. Vice-President can be removed only through a formal
impeachment process similar 2. उप-रा पित को समान औपचा रक महािभयोग ि या के मा म से
to that of President. ही हटाया जा सकता है
3. Dr S. Radhakrishnan was elected as Vice President रा पित के िलए।
continuously for two terms. 3. डॉ. एस. राधाकृ न दो बार लगातार उपरा पित चुने गए।
Which of the above statements is/are correct? उपरो कथनों म से कौन-सा/से सही है /ह?
(a) 3 only
(ए) केवल 3
(b) 1 and 3 only
(बी) केवल 1 और 3
(c) 2 and 3 only
(d) 1 only (सी) केवल 2 और 3
Ans: (a) (डी) केवल 1
Explanation: उ र: (ए)
• Vice-President is not a member, however he/she is the ा ा:
chairman of Council of • उपा सद नहीं होता है , हालां िक वह प रषद का अ होता है
States. There is no exact procedure mentioned in Indian रा । के िलए भारतीय सं िवधान म उ खत कोई सटीक ि या नहीं
Constitution for the
है उपा को हटाना।
removal of Vice President.
3. रा ीय मानवािधकार आयोग (NHRC) के बारे म िन िल खत कथनों
3. Consider the following statements about National Human
पर िवचार कर:
Rights Commission (NHRC):
U-135, Ground Floor, 2nd Floor, Vikas Marg, Baba Complex,
Near Laxmi Nagar Metro Station Gate No. 3&4, Shakarpur, Delhi-92
info@vedanga.co.in
1. It is a Constitutional Body. 1. यह एक सं वैधािनक िनकाय है।
2. It was established in compliance with the Paris Principles of 2.इसकी थापना मानव अिधकारों के पे रस िस ां तों, 1991 के
Human Rights, 1991. अनुपालन म की गई थी।
3. It is composed of a Chairperson and eight other members. 3. इसम एक अ और आठ अ सद होते ह।
Which of the statements given above is/are not correct? ऊपर िदए गए कथनों म से कौन सा/से सही नहीं है /ह?
(a) 1 and 2 only (ए) केवल 1 और 2
(b) 2 and 3 only (बी) केवल 2 और 3
(सी) केवल 1 और 3
(c) 1 and 3 only
(डी) 1, 2 और 3
(d) 1, 2 and 3
उ र: (सी)
Ans: (c)
ा ा: यहाँ िनदश श सही नहीं है !!
Explanation: here the directive word is not correct!!
• एस1: एनएचआरसी 12 अ ू बर, 1993 को मानवािधकार संर ण
• S1: NHRC is a statutory body established on 12th October,
अिधिनयम (पीएचआरए), 1993 के तहत थािपत एक वै धािनक िनकाय
1993 under the Protection of Human Rights Act (PHRA), 1993.
है ।
• S2: NHRC was established in compliance with the Paris
• S2: NHRC की थापना मानवािधकारों के पे रस िस ां तों, 1991 के
Principles of Human Rights, 1991 which were adopted for the
अनुपालन म की गई थी, िजसे मानवािधकारों के चार और संर ण के
promotion and protection of Human Rights and were endorsed
िलए अपनाया गया था और 1993 की महासभा म सं यु रा ारा
by the United Nations at its General Assembly of 1993.
इसका समथन िकया गया था।
• S3: The Commission consists of a Chairperson, five full-time
• S3: आयोग म एक अ , पां च पूणकािलक सद और सात डी ड
Members and seven deemed Members. The statute lays down
सद होते ह। क़ानून आयोग के अ और सद ों की िनयु के
qualifications for the appointment of the Chairperson and
िलए यो ता िनधा रत करता है ।
Members of the Commission.

4. Consider the following statements: 4. िन िल खत कथनों पर िवचार कर:


1. भारत म एक पर एक ही अपराध के िलए एक से अिधक बार
1. In India, a person cannot be prosecuted more than once for the
मुकदमा नहीं चलाया जा सकता है ।
same offence.
2. सं िवधान का अनु े द 21 दोहरे जो खम के खलाफ अिधकार की
2. Article 21 of the Constitution guarantees the right against
गारं टी दे ता है ।
double jeopardy.
उपरो कथनों म से कौन-सा/से सही है /ह?
Which of the above statements is/are correct?
(ए) केवल 1
(a) 1 only (बी) केवल 2
(b) 2 only (सी) 1 और 2 दोनों
(c) Both 1 and 2 (डी) न तो 1 और न ही 2
(d) Neither 1 nor 2 उ र: (ए)
Ans: (a) ा ा:
Explanation: • एक पर एक ही अपराध के िलए एक से अिधक बार मुकदमा
• A person cannot be prosecuted more than once for the same नहीं चलाया जा सकता है ।
offence. • सं िवधान का अनु े द 20(2) दोहरे जो खम के िव अिधकार की
• Article 20(2) of the Constitution guarantees the right against गारं टी दे ता है ।
double jeopardy.
5. रा ीय खा सु र ा अिधिनयम, 2013 के तहत िकए गए ावधानों के
5. With reference to the provisions made under the National सं दभ म,
Food Security Act, 2013,
िन िल खत कथनों पर िवचार कर:
consider the following statements:
1. 'गरीबी रे खा से नीचे (बीपीएल)' की ेणी म आने वाले प रवार ही
1. The families coming under the category of ‘below poverty line
(BPL)’ only are eligible to receive subsidies food grains. स डी खा ा ा करने के पा ह।
2. The eldest woman in a household, of age 18 years or above, 2. राशन काड जारी करने के उ े से घर की सबसे बड़ी मिहला,
shall be the head of the household for the purpose of issuance of िजसकी आयु 18 वष या उससे अिधक है , घर की मु खया होगी।
a ration card. 3. गभवती मिहलाएं और नपान कराने वाली माताएं गभाव था के
3. Pregnant women and lactating mothers are entitled to a ‘take- दौरान और उसके बाद छह महीने के िलए ित िदन 1600 कैलोरी के
home ration’ of 1600 calories per day during pregnancy and for 'टे क-होम राशन' की हकदार ह।
six months ther eafter
ऊपर िदए गए कथनों म से कौन सा/से सही है /ह?
Which of the statement given above is/are correct?
(ए) 1 और 2
(a) 1 and 2
(b) 2 only (बी) केवल 2
(c) 1 and 3 (सी) 1 और 3
(d) 3 only (डी) केवल 3
Ans: (b) उ र: (बी)
Explanation:
ा ा:
• Statement 1: Coverage and entitlement under Targeted Public
• कथन 1: लि त सावजिनक िवतरण के तहत कवरे ज और पा ता
Distribution
System (TPDS): Upto 75% of the rural population and 50% of िस म (टीपीडीएस): ामीण आबादी का 75% तक और शहरी
the urban population will be covered under TPDS, with uniform आबादी का 50% टीपीडीएस के तहत कवर िकया जाएगा, िजसम ित
entitlement of 5 kg per 5 िकलो ाम की समान पा ता होगी। ित माह । केवल
person per month. Not restricted to only BPL families. बीपीएल प रवारों तक ही सीिमत नहीं है ।
• Statement 3: Pregnant women and lactating mothers and
• कथन 3: गभवती मिहलाएं और नपान कराने वाली माताएं और 6
children in the age group of 6 months to 14 years will be entitled
महीने से 14 वष के आयु वग के ब े एकीकृत बाल िवकास से वा
to meals as per prescribed nutritional norms under Integrated
Child Development Services (ICDS) and MidDay Meal (MDM) (आईसीडीएस) और म ा भोजन (एमडीएम) योजनाओं के तहत
schemes. िनधा रत पोषण सं बंधी मानदं डों के अनुसार भोजन के हकदार होंगे।
• The nutritional and feeding norms for supplementary nutrition • भारत सरकार ारा िनधा रत पूरक पोषाहार के िलए पोषण और
as prescribed by GOI are 500 calories and 12-15 grams of आहार के मानदं ड 6 महीने से 6 वष की आयु के ब ों के िलए 500
protein of children between the ages of 6 months to 6 years, 600 कैलोरी और 12-15 ाम ोटीन, गभवती को 600 (1600 नहीं) कैलोरी
(not 1600) calories and 18-20 grams of protein to pregnant and
और 18-20 ाम ोटीन ह। और निसग माताओं और 800 कैलोरी और
nursing mothers and 800 calories and 20-25 gm. of protein to
malnourished children. 20-25 ाम। ोटीन का कुपोिषत ब े।
• Statement 2: Eldest woman of the household of age 18 years or • कथन 2: 18 वष या उससे अिधक आयु की घर की सबसे बु जुग
above to be the मिहला राशन काड जारी करने के उ े से घर के मु खया।
head of the household for the purpose of issuing of ration cards.
6. िन िल खत कथनों पर िवचार कर:
6. Consider the following statements: 1. धान मं ी की आिथक सलाहकार प रषद (EAC-PM) भारत
1. Economic Advisory Council to the Prime Minister (EAC-PM) सरकार, िवशेष प से धान मं ी को आिथक और सं बंिधत मु ों पर
is an independent body constituted to give advice on economic
सलाह दे ने के िलए गिठत एक तं िनकाय है ।
and related issues to the Government of India, specifically to the
Prime Minister. 2. कैिबनेट सिचव आिथक सलाहकार प रषद का पदे न अ होता है
2. The Cabinet Secretary is the ex-officio Chairman of the धान मं ी (EAC-PM) के िलए।
Economic Advisory Council ऊपर िदए गए कथनों म से कौन सा/से सही है /ह?
to the Prime Minister (EAC-PM). (ए) केवल 1
Which of the statements given above is/are correct? (बी) केवल 2
(a) 1 only (सी) 1 और 2 दोनों
(b) 2 only (डी) न तो 1 और न ही 2
(c) Both 1 and 2
उ र: (ए)
(d) Neither 1 nor 2
Ans: (a) ा ा:
Explanation:
• धानमं ी की आिथक सलाहकार प रषद (EAC-PM) एक तं
• Economic Advisory Council to the Prime Minister (EAC-PM)
is an independent सं था है
body constituted to give advice on economic and related issues to आिथक और सं बंिधत मु ों पर सलाह दे ने के िलए गिठत िनकाय
the Government of India, specifically to the Prime Minister. भारत सरकार, िवशेष प से धान मं ी को।
• At present, the composition of EAC-PM is: Dr. Bibek • वतमान म, ईएसी-पीएम की संरचना है : डॉ िबबे क दे बरॉय (अ ),
Debroy (Chairman), ी राकेश मोहन (अंशकािलक सद ), डॉ. सािजद िचनॉय
Shri Rakesh Mohan (Part-Time Member), Dr. Sajjid Chinoy
(अंशकािलक सद ), डॉ. नीलकंठ िम ा (अंशकािलक सद ), ी
(Part-Time Member), Dr. Neelkanth Mishra (Part-Tim Member),
नीलेश शाह (अंशकािलक सद ), ो. टी.टी. राम मोहन (अंशकािलक
Shri Nilesh Shah (Part-Time Member), Prof. T.T. Ram Mohan
(Part-Time Member) and Dr. Poonam Gupta (Part-Time सद ) टाइम सद ) और डॉ. पू नम गु ा (अंशकािलक सद )।
Member).
• The Terms of Reference of EAC-PM include analyzing any • ईएसी-पीएम के सं दभ की शत म धान मं ी ारा सं दिभत िकसी भी
issue, economic or otherwise, referred to it by the Prime Minister आिथक या अ मु े का िव ेषण करना और उ सलाह दे ना,
and advising him thereon, addressing issues of macroeconomic
ापक आिथक मह के मु ों को सं बोिधत करना और धान मं ी को
importance and presenting views thereon to the Prime Minister.
उनके िवचार ु त करना शािमल है । ये या तो ेरणा से या इसके
These could be either suo-motu or on reference from the
Prime Minister or anyone else. They also include attending to सं दभ म हो सकते ह धानमं ी या कोई और। इनम समय-समय पर
any other task as may be desired by the Prime Minister from time धान मं ी ारा वां िछत िकसी भी अ काय को शािमल करना भी
to time. शािमल है ।

7. Consider the following statements about National 7. रा ीय जां च एजसी (एनआईए) के बारे म िन िल खत कथनों पर
Investigation Agency (NIA): िवचार कर:
1. It is the primary counter-terrorist task force of India. 1. यह भारत की ाथिमक आतं कवाद िवरोधी टा फोस है ।
2. It is empowered to deal with the investigation of terror related 2. इसे रा ों से िवशेष अनुमित के िबना रा ों म आतं क सं बंधी
crimes across states without special permission from the states. अपराधों की जां च से िनपटने का अिधकार है ।
3. The founding Director-General of NIA was T. N. Seshan. 3. एनआईए के सं थापक महािनदे शक टी. एन. शेषन थे।
Which of the statements given above is/are correct? ऊपर िदए गए कथनों म से कौन सा/से सही है /ह?
(a) 1 and 2 only (ए) केवल 1 और 2
(b) 2 and 3 only (बी) केवल 2 और 3
(c) 1 and 3 only (सी) केवल 1 और 3
(d) 1, 2 and 3 (डी) 1, 2 और 3
Ans: (a) उ र: (ए)
Explanation: ा ा:
• S3: The founding Director-General of NIA was Radha Vinod • S3: NIA के सं थापक महािनदे शक राधा िवनोद राजू थे, और उ ोंने
Raju, and he served until 31 January 2010. 31 जनवरी 2010 तक से वा की।
• S1 and S2: The National Investigation Agency (NIA) is the • S1 और S2: रा ीय जां च एजसी (NIA) भारत की ाथिमक
primary counterterrorist task force of India. The agency is आतं कवाद िवरोधी टा फोस है । गृ ह मं ालय से िल खत उद् घोषणा
empowered to deal with the investigation of terror related crimes के तहत रा ों से िवशेष अनुमित के िबना रा ों म आतं क सं बंधी
across states without special permission from the states under अपराधों की जां च से िनपटने के िलए एजसी को अिधकार िदया गया है ।
written proclamation from the Ministry of Home Affairs.
8. िवदे शी अंशदान (िविनयमन) अिधिनयम के सं बंध म, िन िल खत म
8. Regarding Foreign Contribution (Regulation) Act, which of से कौन सा/से कथन स है /ह?
the following statements is/are true? 1. एफसीआरए को गृ ह मं ालय ारा लागू िकया जाता है ।
2. एक बार दान िकए जाने के बाद, एफसीआरए पंजीकरण तीन साल
1. FCRA is implemented by the Ministry of Home Affairs.
के िलए वै ध होता है ।
2. Once granted, FCRA registration is valid for three years.
3. अिधिनयम ारा कवर की गई सं थाओं म एक शािमल नहीं
3. The entities covered by the Act does not include an individual. है ।
Select the correct answer using the code below:
नीचे िदए गए कूट का योग कर सही उ र चुिनए:
(a) 1 only (ए) केवल 1
(b) 2 and 3 only (बी) केवल 2 और 3
(c) 1 and 2 only (सी) केवल 1 और 2
(d) 1, 2 and 3 (डी) 1, 2 और 3
ANs: (a) उ र: (ए)
Explanation: ा ा:
• FCRAregulates foreign donations and ensures that such • एफसीआर िवदे शी दान को िनयं ि त करता है और यह सु िनि त
contributions do not adversely affect the internal security of the करता है िक इस तरह के योगदान से दे श की आं त रक सु र ा पर
country. ितकूल भाव न पड़े ।
o The Act, first enacted in 1976 was amended in the year 2010 0 अिधिनयम, िजसे पहली बार 1976 म अिधिनयिमत िकया गया था,
and then 2020. को वष 2010 और िफर 2020 म सं शोिधत िकया गया था।
o Section 5 of the Foreign Contribution (Regulation) Act, िवदे शी अंशदान (िविनयमन) अिधिनयम, 2010 की धारा 5 क सरकार
2010 gives the Union government “unchecked and unbridled को "अिनयं ि त और िनरं कुश श यां " दे ती है िक वह िकसी संगठन
powers” to declare an organisation as being one of political को राजनीितक कृित का सं गठन घोिषत करे और उसे िवदे शों से धन
nature and deny it access to funds from sources abroad. ा करने से मना करे ।
• FCRA is implemented by the Ministry of Home Affairs. • एफसीआरए को गृ ह मं ालय ारा लागू िकया जाता है ।
• Applicability: • यो ता:
o The provisions of the Act apply to the territory of India, to 0 अिधिनयम के ावधान भारत के े म, भारत के नाग रकों पर लागू
citizens of India who may be outside India and to companies or होते ह जो भारत से बाहर हो सकते ह और भारत से बाहर कंपिनयों या
their branches outside India that are registered or incorporated in उनकी शाखाओं पर लागू होते ह जो भारत म पंजीकृत या िनगिमत ह।
India. o अिधिनयम ारा कवर की गई सं थाओं म एक , एक िहं दू
o The entities covered by the Act include an individual, a Hindu अिवभािजत प रवार, एक सं घ या एक पंजीकृत कंपनी शािमल है ।
undivided family, an association, or a registered company. • अनुमोदन कब तक दान िकया जाता है?
• For how long is approval granted? 0 एक बार दान िकए जाने के बाद, एफसीआरए पंजीकरण पां च वष
o Once granted, FCRA registration is valid for five years. के िलए वै ध होता है । एनजीओ से पंजीकरण की समा की तारीख के
NGOs are expected to apply for renewal within six months of the छह महीने के भीतर नवीनीकरण के िलए आवे दन करने की उ ीद है ।
date of expiry of registration. In case of failure to apply for नवीनीकरण के िलए आवे दन करने म िवफल होने की थित म,
renewal, the registration is deemed to have expired, and the NGO पंजीकरण समा हो गया माना जाता है, और एनजीओ अब मं ालय से
is no longer entitled to receive foreign funds or utilise its existing अनुमित के िबना िवदे शी धन ा करने या अपने मौजूदा धन का
funds without permission from the ministry. उपयोग करने का हकदार नहीं है।

9. Consider the following statements regarding POSHAN 9. पोषण अिभयान के सं बंध म िन िल खत कथनों पर िवचार कर।
Abhiyaan.
1. इसे सरकार ारा 8 माच, 2018 को अंतरा ीय मिहला िदवस के
1. It was launched by the government on the occasion of the
अवसर पर लॉ िकया गया था।
International Women’s Day on 8th March, 2018.
2. नीित आयोग पोषण अिभयान के िलए िनगरानी ािधकरण है ।
2. NITI Aayog is the monitoring authority for POSHAN
Abhiyaan. 3. िमशन का ल 2022 तक 0-6 वष आयु वग के ब ों म ं िटं ग को
3. The target of the missionis to eliminate stunting among ख करना है ।
children in the age group 0- 6 years by 2022. उपरो कथनों म से कौन-सा/से सही है /ह?
Which of the above statements is/are correct? (ए) केवल 1 और 3
(a) 1 and 3 only (बी) केवल 1 और 2
(b) 1 and 2 only (सी) केवल 2 और 3
(c) 2 and 3 only
(डी) 1, 2 और 3
(d) 1, 2 and 3
Ans: (b) उ र: (बी)
Explanation: About Poshan Abhiyaan: ा ा: पोषण अिभयान के बारे म:
• The programme seeks to improve nutritional outcomes for • काय म ब ों, गभवती मिहलाओं और नपान कराने वाली
children, pregnant women and lactating mothers. माताओं के िलए पोषण सं बंधी प रणामों म सु धार करना चाहता है ।
• Launched in 2018 with specific targets to be achieved by • 2022 तक ा िकए जाने वाले िविश ल ों के साथ 2018 म लॉ
2022.
िकया गया।
• It aims to reduce:
• इसका उ े कम करना है :
o Stunting and wasting by 2% a year (total 6% until 2022)
among children. o ब ों म ं िटं ग और वे ं ग ित वष 2% (2022 तक कुल 6%)।
o Anaemia by 3% a year (total 9%) among children, adolescent ब ों, िकशोर लड़िकयों और गभवती मिहलाओं और नपान कराने
girls and pregnant women and lactating mothers. वाली माताओं म ित वष 3% (कुल 9%) एनीिमया।
o The target of the mission is to bring down stunting among o िमशन का ल 2022 तक 0-6 वष आयु वग के ब ों म ं िटं ग को
children in the age group 0-6 years from 38.4% to 25% by 2022.
38.4% से 25% तक कम करना है ।
• NITI Aayog has played a critical role in shaping the POSHAN
• नीित आयोग ने पोषण अिभयान को आकार दे ने म मह पूण भू िमका
Abhiyaan. NITI
Aayog has been entrusted with the task of closely monitoring िनभाई है । नीित
the POSHANAbhiyaan and undertaking periodic evaluations. आयोग को पोषण अिभयान की बारीकी से िनगरानी करने और समय-
समय पर मू ां कन करने का काम सौंपा गया है ।
10. Consider the following statements about the Mid-Day meal
scheme: 10. म ा भोजन योजना के बारे म िन िल खत कथनों पर िवचार कर:
1. The scheme guarantees one meal to all children in government
1. यह योजना सरकारी और सहायता ा ू लों और सम िश ा के
and aided schools and madarsas supported under Samagra
तहत समिथत मदरसों म सभी ब ों को एक भोजन की गारं टी दे ती है ।
Shiksha.
2. The Scheme comes under the Ministry of Consumer Affairs, 2. यह योजना उपभो ा मामले, खा और सावजिनक िवतरण मं ालय
Food and Public Distribution. के अंतगत आती है ।
Which of the statements given above is/are correct? ऊपर िदए गए कथनों म से कौन सा/से सही है /ह?
(a) 1 only (ए) केवल 1
(b) 2 only (बी) केवल 2
(c) Both 1 and 2 (सी) 1 और 2 दोनों
(d) Neither 1 nor 2 (डी) न तो 1 और न ही 2
Ans: (a) उ र: (ए)
Explanation: About the Mid-Day meal scheme: ा ा: म ा भोजन योजना के बारे म:
• The scheme guarantees one meal to all children in government • यह योजना सरकारी और सहायता ा ू लों और सम िश ा के
and aided schools and madarsas supported under Samagra तहत समिथत मदरसों म सभी ब ों को एक भोजन की गारं टी दे ती है ।
Shiksha. • आठवीं क ा तक के छा ों को साल म कम से कम 200 िदनों के
• Students up to Class VIII are guaranteed one nutritional cooked
िलए एक पका आ पोषाहार भोजन की गारं टी दी जाती है ।
meal at least 200 days in a year.
• The Scheme comes under the Ministry of HRD. • यह योजना मानव सं साधन िवकास मं ालय के अंतगत आती है ।
• It was launched in the year 1995 as the National Programme of • इसे वष 1995 म ाथिमक िश ा के िलए पोषण सं बंधी सहायता के
Nutritional Support toPrimary Education (NP – NSPE), a रा ीय काय म (एनपी-एनएसपीई) के प म शु िकया गया था, जो
centrally sponsored scheme. In एक क ायोिजत योजना है । म 2004 म, इस योजना को िमड डे मील
2004, the scheme was relaunched as the Mid Day Meal Scheme. योजना के प म िफर से शु िकया गया था।
• The Scheme is also covered by the National Food Security Act,
• यह योजना रा ीय खा सु र ा अिधिनयम, 2013 के अंतगत भी आती
2013
है
11. Regarding Money Bill, which of the following statements is
not correct? 11. धन िवधे यक के सं बंध म, िन िल खत म से कौन सा कथन सही
(a) A bill shall be deemed to be money bill if it contains only नहीं है ?
provisions related to imposition, abolition, remission, alteration (ए) एक िबल को मनी िबल माना जाएगा यिद इसम केवल िकसी कर
or regulation of any tax.
के लगाने , उ ूलन, छूट, प रवतन या िविनयमन से सं बंिधत ावधान ह।
(b) A Money Bill has provisions for the custody of the
(बी) एक मनी िबल म भारत के समेिकत िनिध या भारत के आक क
Consolidated Fund of India or the Contingency Fund of India.
(c) A Money Bill is concerned with appropriation of moneys out िनिध की िहरासत के ावधान ह।
of the Contingency Fund of India. (c) धन िवधे यक का सं बंध भारत की आक कता िनिध से धन के
(d) A Money Bill deals with the regulation of borrowing of िविनयोग से है ।
money or giving of any guarantee by the government of India. (d) धन िवधे यक भारत सरकार ारा धन उधार लेने या कोई गारं टी दे ने
Ans: (c) के िनयमन से सं बंिधत है ।
Explanation: A110. Definition of Money Bill
उ र: (सी)
• (1) For the purposes of this Chapter, a Bill shall be deemed to
be a Money Bill if it contains only provisions dealing with all or ा ा: A110. धन िवधे यक की प रभाषा
any of the following matters, namely • (1) इस अ ाय के योजनों के िलए, एक िवधे यक को धन िवधे यक
• (a) the imposition, abolition, remission, alteration or regulation माना जाएगा यिद इसम केवल िन िल खत सभी या िकसी भी मामले से
of any tax; सं बंिधत ावधान शािमल ह, अथात्
• (b) the regulation of the borrowing of money or the giving of • (ए) िकसी कर का अिधरोपण, उ ू लन, छूट, प रवतन या िविनयमन;
any guarantee by the Government of India, or the amendment of
• (बी) भारत सरकार ारा पैसे उधार ले ने या कोई गारं टी दे ने का
the law with respect to any financial obligations undertaken or to
िविनयमन, या भारत सरकार ारा िकए गए या िकए जाने वाले िकसी
be undertaken by the Government of India;
• (c) the custody of the consolidated Fund or the Contingency भी िव ीय दािय ों के सं बंध म कानून म सं शोधन;
Fund of India, the payment of moneys into or the withdrawal of • (सी) भारत की समेिकत िनिध या आक कता िनिध की अिभर ा,
moneys from any such Fund; ऐसी िकसी िनिध म धन का भु गतान या उससे धन की िनकासी;
• (d) the appropriation of moneys out of the consolidated Fund of • (घ) भारत की सं िचत िनिध (भारत की आक कता िनिध नहीं) म से
India (not Contingency Fund of India). So, C is incorrect.
धन का िविनयोग। अत: C गलत है ।

12. Consider the following statements:


12. िन िल खत कथनों पर िवचार कर:
1. Speaker of the legislative assembly shall vacate his/her office
1. िवधान सभा के अ िवधानसभा के सद नहीं रहने पर अपना
of he/she ceases to be a member of the assembly
कायालय खाली कर दगे
2. Whenever the legislative assembly is dissolved the speaker
2. जब भी िवधान सभा भं ग की जाती है तो अ अपना कायालय
shall vacate his/her office immediately
तु रंत खाली कर दे गा
Which of the statements given above is/are correct? ऊपर िदए गए कथनों म से कौन सा/से सही है /ह?
(a) 1 only (ए) केवल 1
(b) 2 only
(बी) केवल 2
(c) Both 1 and 2
(सी) 1 और 2 दोनों
(d) Neither 1 not 2
Ans: (a) (डी) न तो 1 नहीं 2
Explanation: उ र: (ए)
• Usually, the speaker remains in office during the life of the ा ा:
assembly. However, he vacates his office earlier in any of the • आम तौर पर ीकर िवधानसभा के कायकाल के दौरान अपने पद
following cases: पर बना रहता है । हालाँ िक, वह िन म से िकसी भी मामले म पहले
o If he ceases to be a member of the assembly
अपना कायालय छोड़ दे ता है :
o If he resigns by writing to the deputy speaker and;
o If he is removed by a resolution passed by a majority of all the o यिद वह िवधानसभा का सद नहीं रह जाता है
then members of the assembly. Such a resolution can be moved o यिद वह िड ी ीकर को िल खत प से इ ीफा दे ता है और;
only after giving 14 days advance notice o यिद उसे िवधानसभा के सभी त ालीन सद ों के ब मत से पा रत
• Statement 2 is incorrect because the Speaker holds office from ाव ारा हटाया जाता है । ऐसा सं क 14 िदन की अि म सू चना
the date of her election till immediately before the first meeting दे कर ही पेश िकया जा सकता है
of the Legislative assembly after the dissolution of the one to
• कथन 2 गलत है ोंिक अ अपने चु नाव की तारीख से िवधान
which she was elected. She is eligible for re-election. On the
dissolution of the Legislative assembly, although the Speaker सभा की पहली बै ठक से ठीक पहले तक पद धारण करती है, िजसके
ceases to be a member of the House, she does not vacate her िलए वह िनवािचत ई थी, उसके िवघटन के बाद। वह िफर से चुनाव के
office. िलए पा है । िवधान सभा के िवघटन पर, हालां िक अ सदन का
सद नहीं रहता है , वह अपना कायालय खाली नही ं करती है ।
13. 43. Regarding Parliamentary Committees, which of the
following statements is/are correct? 13. 43. सं सदीय सिमितयों के सं बंध म, िन िल खत म से कौन सा/से
1. Committees which are appointed or elected by the House or कथन सही है /ह?
nominated by the Speaker 1. सिमितयां जो सदन ारा िनयु या चुनी जाती ह या अ ारा
2. Committees which works under the direction of the Speaker नािमत की जाती ह
and presents its report to the House or to the Speaker and the 2. सिमितयाँ जो अ के िनदशन म काम करती ह और अपनी रपोट
Secretariat सदन या अ और सिचवालय को ु त करती ह
3. Committees which are established by the Prime Minister as 3. सिमितयाँ जो धानमं ी ारा समय की आव कता और प र थित
per the exigencies of the time and needs of the situation की आव कता के अनुसार थािपत की जाती ह
Select the correct answer using the code below: नीचे िदए गए कूट का योग कर सही उ र चुिनए:
(a) 1 only (ए) केवल 1
(b) 1 and 2 only (बी) केवल 1 और 2
(c) 2 and 3 only (सी) केवल 2 और 3
(d) 1, 2 and 3 (डी) 1, 2 और 3
Ans: (b) उ र: (बी)
Explanation: ा ा:
• Parliamentary Committees: • सं सदीय सिमितयां :
o The Lok Sabha website describes a parliamentary committee as 0 लोकसभा की वे बसाइट एक सं सदीय सिमित को "सिमित जो सदन
a “committee which is appointed or elected by the House or ारा िनयु या िनवािचत या अ ारा नािमत िकया जाता है और जो
nominated by the Speaker and which works under the direction अ के िनदशन म काम करती है और सदन या अ और
of the Speaker and presents its report to the House or to the सिचवालय को अपनी रपोट ु त करती है" के प म विणत करती
Speaker and the Secretariat”. है । .
• Cabinet Committees: • कैिबनेट सिमितयां :
o The Prime Minister sets up different cabinet committees with 0 धानमं ी कैिबनेट के चयिनत सद ों के साथ िविभ कैिबनेट
selected members of the Cabinet and assigns specific functions सिमितयों का गठन करते ह और इन सिमितयों को िविश काय सौंपते
to these committees. ह।

14. Consider the following statements: 14. िन िल खत कथनों पर िवचार कीिजएः


1. भारत के मूल सं िवधान म या िमक सं शोधनों म कॉलेिजयम णाली
1. There is no mention of the Collegium system either in the
का कोई उ ेख नही ं है ।
original Constitution of India or in successive amendments.
2. High Court judges are recommended by a Collegium system 2. उ ायालय के ायाधीशों की िसफा रश एक कॉलेिजयम णाली
comprising the CJI and four senior-most judges. ारा की जाती है िजसम CJI और चार व र तम ायाधीश शािमल होते
Which of the given above statements is/are correct? ह।
(a) 1 only
ऊपर िदए गए कथनों म से कौन-सा/से सही है /ह?
(b) 2 only
(ए) केवल 1
(c) Both 1 and 2
(बी) केवल 2
(d) Neither 1 nor 2
(सी) 1 और 2 दोनों
Ans: (a)
Explanation: (डी) न तो 1 और न ही 2
• S1: The Collegium of judges is the Supreme Court’s invention. उ र: (ए)
It does not figure in the Constitution, which says judges of the ा ा:
Supreme Court and High Courts are appointed by the President • S1: ायाधीशों का कॉलेिजयम सव ायालय का आिव ार है।
and speaks of a process of consultation. यह सं िवधान म शािमल नहीं है , जो कहता है िक सव ायालय और
• S2: High Court judges are recommended by a Collegium
उ ायालयों के ायाधीशों को रा पित ारा िनयु िकया जाता है
comprising the CJI and two senior-most judges.
o The proposal, however, is initiated by the Chief Justice of the और परामश की ि या की बात करता है ।
High Court concerned in consultation with two senior-most • S2: उ ायालय के ायाधीशों की िसफा रश CJI और दो
colleagues. The recommendation is sent to the Chief Minister, व र तम ायाधीशों वाले कॉलेिजयम ारा की जाती है ।
who advises the Governor to send the proposal to the Union Law हालां िक, ाव सं बंिधत उ ायालय के मु ायाधीश ारा दो
Minister. व र तम सहयोिगयों के परामश से शु िकया जाता है । अनुशंसा
मु मं ी को भे जी जाती है, जो रा पाल को सलाह दे ता है िक वह
15.. Consider the following statements about Dr Rajendra
ाव के ीय कानून मं ी को भेजे।
Prasad:
1. He was the first president of independent India.
2. He served as the Food and Agriculture Minister in the central 15.. डॉ. राजे साद के बारे म िन िल खत कथनों पर िवचार कीिजएः
1. वे तं भारत के थम रा पित थे।
government after the 1946 elections.
2. उ ोंने 1946 के चुनावों के बाद क सरकार म खा और कृिष मं ी
3. He served as the President of the Constituent Assembly of
के प म काय िकया।
India.
3. उ ोंने भारत की सं िवधान सभा के अ के प म काय िकया।
Which of the statements given above is/are correct?
ऊपर िदए गए कथनों म से कौन सा/से सही है /ह?
(a) 1 and 2 only
(ए) केवल 1 और 2
(b) 2 and 3 only (बी) केवल 2 और 3
(c) 1 and 3 only (सी) केवल 1 और 3
(d) 1, 2 and 3 (डी) 1, 2 और 3
Ans: (d) उ र: (डी)
Explanation: ा ा:
• Awards in public administration in the field of academic • भारत के पहले रा पित की ृ ित म अकादिमक उ ृ ता के े म
excellence, in memory of the first President of India. लोक शासन म पुर ार।
• Dr Rajendra Prasad was the first president of independent • डॉ. राज साद तं भारत के पहले रा पित थे। वह एक भारतीय
India. He was an Indian Freedom Activist, Lawyer, and Scholar तं ता कायकता, वकील और िव ान भी थे। उ ोंने 1946 के चुनावों
too. He served as the Food and Agriculture Minister in the के बाद क सरकार म खा और कृिष मं ी के प म काय िकया।
central government after the 1946 elections. He also served as उ ोंने भारत की संिवधान सभा के अ के प म भी काय िकया।
the President of the Constituent Assembly of India. • सािह क काय
• LITERARY WORKS चंपारण म स ा ह (1922)
o Satyagraha at Champaran (1922) ओ इं िडया िडवाइडे ड (1946)
o India Divided (1946) o आ कथा (1946) बां कीपुर जेल म 3 साल की जेल की अविध के
o Atmakatha (1946) his autobiography written during his 3-year दौरान िलखी गई उनकी आ कथा
prison term in Bankipur Jail महा ा गां धी और िबहार, कुछ याद (1949)
o Mahatma Gandhi and Bihar, Some Reminiscences (1949) ओ बापू के कदमों म (1954)
o Bapu Ke Kadmon Mein (1954) तं ता के बाद से (1960)
o Since Independence (1960)
16. िस ेया िसं घल मामला हाल ही म खबरों म था, िन िल खत म
16. The famous Shreya Singhal case was in news recently, is से िकससे सं बंिधत है?
related to which of the following?
(a) Capital punishment (ए) मृ ुदंड
(b) Disqualification of legislators (b) िवधायकों की अयो ता
(c) Model code of conduct (सी) आदश आचार सं िहता
(d) None of the above (डी) उपयु म से कोई नही ं
Ans: (d) उ र: (डी)
Explanation:
ा ा:
• What is Section 66A?
o Section 66A defines the punishment for sending “offensive” • धारा 66ए ा है ?
messages through a computer or any other communication 0 धारा 66ए एक कं ूटर या िकसी अ सं चार उपकरण जैसे मोबाइल
device like a mobile phone or a tablet. फोन या टै बलेट के मा म से "आपि जनक" सं देश भे जने के िलए
o A conviction can fetch a maximum of three years in jail and a सजा को प रभािषत करती है ।
fine. 0 दोष िस होने पर अिधकतम तीन वष की जेल और जुमाना हो
o It empowered police to make arrests over what policemen, in
सकता है ।
terms of their subjective discretion, could construe as “offensive”
0 इसने पुिलस को यह अिधकार िदया िक पुिलसकम अपने
or “menacing” or for the purposes of causing annoyance,
inconvenience, etc. परक िववे क के अनुसार, "अपमानजनक" या "खतरनाक" या
• Shreya Singhal case: झुंझलाहट, असु िवधा, आिद के उ े ों के िलए ा कर सकते ह, इस
o The Supreme Court had in its judgment in the Shreya Singhal पर िगर ारी कर सकते ह।
case struck down Section 66A. • ेया िसं घल मामला:
0 सव ायालय ने ेया िसं घल मामले म अपने फैसले म धारा 66ए
17. Which of the following is/are the aims of “Digital India” Plan
को र कर िदया था।
of the Government of India?
1. Formation of India’s own Internet companies like China did.
2. Establish a policy framework to encourage overseas 17. िन िल खत म से कौन-सा/से भारत सरकार की "िडिजटल इं िडया"
multinational corporations that collect Big Data to build their योजना का/के उ े ह/ह?
large data centres within our national geographical boundaries. 1. चीन की तरह भारत की अपनी इं टरनेट कंपिनयों का गठन िकया।
2. हमारी रा ीय भौगोिलक सीमाओं के भीतर अपने बड़े डे टा क ों का
3. Connect many of our villages to the Internet and bring Wi-Fi
िनमाण करने के िलए िबग डे टा एक करने वाले िवदे शी ब रा ीय
to many of our schools, public places and major tourist centres.
िनगमों को ो ािहत करने के िलए एक नीितगत ढां चा थािपत कर।
Select the correct answer using the code given below:
3. हमारे कई गाँ वों को इं टरनेट से जोड़ और हमारे कई ू लों,
(a) 1 and 2 only
सावजिनक थानों और मुख पयटन क ों म वाई-फाई लाएँ ।
(b) 3 only
नीचे िदए गए कूट का योग कर सही उ र चुिनए:
(c) 2 and 3 only (ए) केवल 1 और 2
(d) 1, 2 and 3 (बी) केवल 3
Ans: (b) (सी) केवल 2 और 3
Explanation:’ (डी) 1, 2 और 3
• Only statement 3 is correct उ र: (बी)
• The Digital India programme is centred on three key vision ा ा:'
areas: • केवल कथन 3 सही है
• Digital Infrastructure as a Core Utility to Every Citizen • िडिजटल इं िडया काय म तीन मुख ि े ों पर कि त है :
• Governance and Services on Demand • ेक नाग रक के िलए एक मुख उपयोिगता के प म िडिजटल
• Digital Empowerment of Citizens अवसं रचना
• मां ग पर शासन और से वाएं
18. In India, the population Census is a • नाग रकों का िडिजटल अिधका रता
(a) Union subject
(b) State subject 18. भारत म जनसं ा जनगणना है
(c) Concurrent subject (ए) सं घ िवषय
(d) Reserved subject (बी) रा िवषय
Ans: (a) (सी) समवत िवषय
Explanation: (डी) आरि त िवषय
• The authority to conduct Census in India is derived from उ र: (ए)
Article 246 of the ा ा:
Constitution of India. This article empowers the Parliament to • भारत म जनगणना करने का अिधकार अनु े द 246 से िलया गया है
भारत का सं िवधान। यह अनु े द सं सद को अिधकार दे ता है
make laws with respect to any of the matters enumerated in the सातवी ं अनुसूची म सू ची-I म उ खत िकसी भी मामले के संबंध म
List-I in the Seventh Schedule referred to as 'Union List" and कानून बनाना िजसे 'सं घ सू ची' और 'जनगणना' िवषय के प म
the subject 'Census'.
सं दिभत िकया गया है ।

19. Consider the following statements regarding National


Commission for Backward Classes (NCBC). 19. रा ीय िपछड़ा वग आयोग (एनसीबीसी) के सं बंध म िन िल खत
1. National Commission for Backward Classes is a non- कथनों पर िवचार कर।
constitutional body under the Ministry of Social Justice and 1. रा ीय िपछड़ा वग आयोग सामािजक ाय और अिधका रता
Empowerment. मं ालय के तहत एक गै र-सं वैधािनक िनकाय है ।
2. The commission considers inclusions in and exclusions from 2. आयोग नौकरी म आर ण के उ े से िपछड़े के प म
the lists of communities notified as backward for the purpose of अिधसू िचत समुदायों की सू ची म शािमल और बिह रण पर िवचार
job reservations.
करता है ।
3. The commission have the same powers as a Civil Court.
3. आयोग के पास िसिवल कोट के समान श यां ह।
Which of the above statements is/are correct?
उपरो कथनों म से कौन-सा/से सही है /ह?
(a) 1 and 2 only
(b) 1 and 3 only (ए) केवल 1 और 2
(c) 2 and 3 only (बी) केवल 1 और 3
(d) 1, 2 and 3 (सी) केवल 2 और 3
Ans: (c) (डी) 1, 2 और 3
Explanation:
उ र: (सी)
• National Commission for Backward Classes is a
ा ा:
constitutional body (123rd constitutional amendment bill 2017
and 102nd amendment 2018 in constitution to make it • रा ीय िपछड़ा वग आयोग एक सं वैधािनक िनकाय है (इसे सं वैधािनक
constitutional body) (Article 338B of the Indian Constitution) िनकाय बनाने के िलए सं िवधान म 123वां सं वैधािनक सं शोधन िवधे यक
under Ministry of Social Justice and Empowerment established 2017 और 102वां सं शोधन 2018) (भारतीय सं िवधान का अनु े द
on 14 August 1993. It was constituted pursuant to the provisions 338B) सामािजक ाय और अिधका रता मं ालय के तहत 14 अग
of the National Commission for Backward Classes Act, 1993. 1993 को थािपत िकया गया था। यह रा ीय िपछड़ा वग आयोग
• The commission was the outcome of Indra Sawhney & Ors.
अिधिनयम, 1993 के ावधानों के अनुसार गिठत िकया गया था।
Vs. Union of India.
• The commission considers inclusions in and exclusions from • आयोग इं ा साहनी और अ का प रणाम था। बनाम भारत संघ।
the lists of communities notified as backward for the purpose of • आयोग नौकरी म आर ण के उ े से िपछड़े के प म अिधसू िचत
job reservations and tenders the needful advice to the Central समुदायों की सू ची म शािमल करने और बाहर करने पर िवचार करता
Government as per Section 9(1) of the NCBC Act, 1993. है और एनसीबीसी अिधिनयम, 1993 की धारा 9(1) के अनुसार क
Similarly, the states have also constituted commissions for सरकार को आव क सलाह दे ता है । इसी कार, रा ों ने के िलए
BC's. The National Commission for Backward Classes, National
आयोगों का भी गठन िकया
Commission for Scheduled Castes as well as National
ईसा पूव। रा ीय िपछड़ा वग आयोग, रा ीय अनुसूिचत जाित आयोग
Commission for Scheduled Tribes have the same powers as a
Civil Court. और रा ीय अनुसूिचत जनजाित आयोग के पास िसिवल कोट के समान
अिधकार ह।
20. 84. Consider the following statements about the Monkeypox
disease: 20. 84. मंकीपॉ रोग के बारे म िन िल खत कथनों पर िवचार कर:
1. It is a viral zoonotic disease with symptoms similar to those 1. यह एक वायरल जूनोिटक रोग है िजसके ल ण पूव म चेचक के
seen in the past in smallpox patients. रोिगयों म दे खे गए ल णों के समान ह।
2. It is usually a self-limited disease with the symptoms lasting 2. यह आमतौर पर 2 से 4 स ाह तक चलने वाले ल णों के साथ एक
from 2 to 4 weeks. -सीिमत बीमारी है ।
Which of the statements given above is/are correct? ऊपर िदए गए कथनों म से कौन सा/से सही है /ह?
(a) 1 only (ए) केवल 1
(b) 2 only (बी) केवल 2
(c) Both 1 and 2 (सी) 1 और 2 दोनों
(डी) न तो 1 और न ही 2
(d) Neither 1 nor 2
उ र: (सी)
Ans: (c)
ा ा:
Explanation:
• मंकीपॉ एक वायरल ज़ूनोिसस (मनु ों म फैलने वाला वायरस) है
• Monkeypox is a viral zoonosis (a virus transmitted to humans
from animals) with symptoms similar to those seen in the past in जानवरों से ) चेचक के रोिगयों म अतीत म दे खे गए ल णों के समान,
smallpox patients, although it is clinically less severe. हालां िक यह िचिक कीय प से कम गं भीर है ।
• Monkeypox is usually a self-limited disease with the symptoms
• मंकीपॉ आमतौर पर 2 से 4 स ाह तक रहने वाले ल णों के साथ
lasting from 2 to 4 weeks. Severe cases can occur. In recent
एक -सीिमत बीमारी है । गं भीर मामले हो सकते ह। हाल के िदनों म,
times, the case fatality ratio has been around 3–6%.
मृ ु दर अनुपात लगभग 3-6% रहा है ।
21. The national motto of India, ‘Satyameva Jayate’ inscribed
below the Emblem of India is taken from: 21. भारत के तीक के नीचे अंिकत भारत का रा ीय आदश वा ,
(a) Katha Upanishad 'स मेव जयते ' कहाँ से िलया गया है :
(b) Chandogya Upanishad (ए) कथा उपिनषद
(c) Aitareya Upanishad (बी) चंदो उपिनषद
(d) Mundaka Upanishad (c) ऐतरे य उपिनषद
Ans: (d)
(डी) मुंडका उपिनषद
Explanation:
उ र: (डी)
• Satyameva Jayate ('Truth alone triumphs') is a part of a
mantra from the Hindu scripture Mundaka Upanishad. ा ा:
Following the independence of India, it was adopted as the • स मेव जयते ('स की ही जीत होती है ') िहं दू शा मुंडक
national motto of India on 26 January 1950, the day India उपिनषद के एक मं का एक िह ा है । भारत की तं ता के बाद,
became a republic. इसे 26 जनवरी 1950 को भारत के रा ीय आदश वा के प म
अपनाया गया, िजस िदन भारत
22. “To abide by the Constitution and respect its ideals and
गणतं बन गया।
institutions, the National Flag and the National Anthem ” is a
provision made in the
(a) Preamble of the Constitution 22. "सं िवधान का पालन करने और उसके आदश और सं थाओं,
रा ीय ज और रा गान का स ान करने के िलए" िकसम ावधान
(b) Directive Principles of State Policy
िकया गया है?
(c) Fundamental Rights
(ए) सं िवधान की ावना
(d) Fundamental Duties (बी) रा नीित के िनदशक िस ां त
Ans: (d)
(सी) मौिलक अिधकार
Explanation:
(डी) मौिलक कत
• Constitutional & Statutory Provisions regarding National
उ र: (डी)
Flag of India:
ा ा:
• Art 51A (a) –To abide by the Constitution and respect its ideals
• भारत के रा ीय ज के सं बंध म सं वैधािनक और वै धािनक ावधान:
and institutions, the National Flag and the National Anthem.
• कला 51ए (ए) -सं िवधान का पालन करना और उसके आदश और
• Statutes Governing Use of Flag:
सं थाओं, रा ीय ज और रा गान का स ान करना।
o Emblems and Names (Prevention of Improper Use) Act, 1950.
• ज के उपयोग को िनयं ि त करने वाले क़ानून:
o Prevention of Insults to National Honor Act, 1971.
0 सं तीक और नाम (अनुिचत योग िनवारण) अिधिनयम, 1950।
o रा ीय स ान अपमान िनवारण अिधिनयम, 1971।
23. With reference to ‘National Institution for Transforming
India’ (NITI Aayog), consider the following statements: 23. 'नेशनल इं ी ूशन फॉर टां सफॉिमग इं िडया' (नीित आयोग) के
1. It was formed via a resolution of the Union Cabinet on 1 सं दभ म िन िल खत कथनों पर िवचार कर:
January 2014. 1. इसका गठन 1 जनवरी 2014 को क ीय मंि मंडल के एक ाव
2. It comprises all the state Governors and Chief Ministers, along के मा म से िकया गया था।
with the Chief 2. इसम मु मं ी के साथ-साथ सभी रा के रा पाल और मु मं ी
Ministers of Delhi and Puducherry, Lieutenant Governors of all शािमल होते ह
UTs, and a vice chairman nominated by the Prime Minister. िद ी और पुडुचेरी के मं ी, सभी सं घ शािसत दे शों के उपरा पाल
Which of the given above statements is/are correct? और धान मं ी ारा नािमत एक उपा ।
(a) 1 only ऊपर िदए गए कथनों म से कौन-सा/से सही है /ह?
(b) 2 only (ए) केवल 1
(c) Both 1 and 2 (बी) केवल 2
(d) Neither 1 nor 2 (सी) 1 और 2 दोनों
Ans: (d) (डी) न तो 1 और न ही 2
Explanation: उ र: (डी)
ा ा:
• S1: The NITI Aayog is a policy think tank of the Government S1: नीित आयोग भारत सरकार का एक नीित िथंक टक है, िजसे
of India, established in 2015 to replace the Planning योजना आयोग को बदलने के िलए 2015 म थािपत िकया गया था।
Commission. • S2: इसम िद ी और पुडुचेरी के मु मंि यों, सभी सं घ शािसत
• S2: It comprises all the state Chief Ministers (not दे शों के उपरा पालों और धान मं ी ारा नािमत एक उपा के
Governors), along with the Chief Ministers of Delhi and साथ-साथ सभी रा ों के मु मं ी (रा पाल नहीं) शािमल ह।
Puducherry, Lieutenant Governors of all UTs, and a vice-
chairman nominated by the Prime Minister. 24. िवदे शी अंशदान (िविनयमन) अिधिनयम के सं बंध म, िन िल खत म
से कौन सा कथन स है / ह?
24. Regarding Foreign Contribution (Regulation) Act, which of 1. एफसीआरए को गृ ह मं ालय ारा लागू िकया जाता है ।
the following statements is/are true? 2. एक बार दान िकए जाने के बाद, एफसीआरए पंजीकरण तीन साल
1. FCRA is implemented by the Ministry of Home Affairs. के िलए वै ध होता है ।
2. Once granted, FCRA registration is valid for three years. 3. अिधिनयम ारा कवर की गई सं थाओं म एक शािमल नहीं
3. The entities covered by the Act does not include an individual. है ।
Select the correct answer using the code below: नीचे िदए गए कूट का योग कर सही उ र चुिनए:
(a) 1 only (ए) केवल 1
(b) 2 and 3 only (बी) केवल 2 और 3
(c) 1 and 2 only (सी) केवल 1 और 2
(डी) 1, 2 और 3
(d) 1, 2 and 3
उ र: (ए)
Ans: (a)
ा ा:
Explanation:
• एफसीआर िवदे शी दान को िनयं ि त करता है और यह सु िनि त
• FCRAregulates foreign donations and ensures that such
करता है िक इस तरह के योगदान से दे श की आं त रक सु र ा पर
contributions do not adversely affect the internal security of the
ितकूल भाव न पड़े ।
country.
• अिधिनयम, िजसे पहली बार 1976 म अिधिनयिमत िकया गया था, को
• The Act, first enacted in 1976 was amended in the year 2010
वष 2010 और िफर 2020 म सं शोिधत िकया गया था।
and then 2020.
• िवदे शी अंशदान (िविनयमन) अिधिनयम, 2010 की धारा 5 क
• Section 5 of the Foreign Contribution (Regulation) Act,
सरकार को एक राजनीितक कृित के सं गठन के प म घोिषत करने
2010 gives the Union government “unchecked and unbridled
के िलए "अिनयं ि त और बे लगाम श यां " दे ती है और इसे िवदे शों के
powers” to declare an organisation as being one of political
ोतों से धन तक प ं च से वं िचत करती है ।
nature and deny it access to funds from sources abroad.
• एफसीआरए को गृ ह मं ालय ारा लागू िकया जाता है ।
• FCRA is implemented by the Ministry of Home Affairs.
• यो ता:
• Applicability:
एमसी ू (आरटीएम) सं कलन के मा म से सं शोधन (जुलाई - 2022)
Revision Through MCQs (RTM) Compilation (July -
• अिधिनयम के ावधान भारत के े पर लागू होते ह, भारत के
2022)
नाग रकों पर लागू होते ह जो भारत के बाहर हो सकते ह और उन
• The provisions of the Act apply to the territory of India, to
कंपिनयों या उनकी शाखाओं पर लागू होते ह जो भारत म पंजीकृत या
citizens of India who may be outside India and to companies or
िनगिमत ह।
their branches outside India that are registered or incorporated in
• अिधिनयम ारा कवर की गई सं थाओं म एक , एक िहं दू
India.
अिवभािजत प रवार, एक सं घ या एक पंजीकृत कंपनी शािमल है ।
• The entities covered by the Act include an individual, a Hindu
• अनुमोदन कब तक दान िकया जाता है?
undivided family, an association, or a registered company.
• एक बार दान िकए जाने के बाद, एफसीआरए पंजीकरण पां च साल
• For how long is approval granted?
के िलए वै ध होता है । एनजीओ से पंजीकरण की समा की तारीख के
• Once granted, FCRA registration is valid for five years.
छह महीने के भीतर नवीनीकरण के िलए आवे दन करने की उ ीद है ।
NGOs are expected to apply for renewal within six months of the
नवीनीकरण के िलए आवे दन करने म िवफलता के मामले म, पंजीकरण
date of expiry of registration. In case of failure to apply for
समा हो गया माना जाता है, और
renewal, the registration is deemed to have expired, and
एनजीओ अब मं ालय से अनुमित के िबना िवदे शी धन ा करने या
the NGO is no longer entitled to receive foreign funds or utilise
अपने मौजूदा धन का उपयोग करने का हकदार नहीं है ।
its existing funds without permission from the ministry.
25. िन िल खत कथनों पर िवचार कीिजएः
25. Consider the following statements:
1. 73व सं वैधािनक सं शोधन अिधिनयम ने शहरी थानीय िनकायों
1. The 73th constitutional amendment act mandated the setting
(यू एलबी) को श यों की थापना और ह ां तरण को अिनवाय कर
up and devolution of powers to Urban local bodies (ULBs).
िदया।
2. Urban governance is part of the state list under the 2. शहरी शासन सं िवधान के तहत रा सू ची का िह ा है ।
Constitution. ऊपर िदए गए कथनों म से कौन सा/से सही है /ह?
Which of the statements given above is/are correct?
(a) 1 only (ए) केवल 1
(b) 2 only (बी) केवल 2
(c) Both 1 and 2 (सी) 1 और 2 दोनों
(d) Neither 1 nor 2 (डी) न तो 1 और न ही 2
Ans: (b) उ र: (बी)
Explanation:
ा ा:
• The 74th Constitution Amendment Act was passed in 1992
mandating the setting up and devolution of powers to urban • 74वां सं िवधान सं शोधन अिधिनयम 1992 म शहरी थानीय िनकायों
local bodies (ULBs) as the lowest unit of governance in cities (यू एलबी) को शहरों और क ों म शासन की सबसे िनचली इकाई के
and towns. प म थािपत करने और श यों के ह ां तरण को अिनवाय करने
• Constitutional provisions were made for ULBs’ fiscal के िलए पा रत िकया गया था।
empowerment. • यू एलबी के िव ीय सश करण के िलए सं वैधािनक ावधान िकए
o Urban governance is part of the state list under the Constitution गए।
o शहरी शासन सं िवधान के तहत रा सू ची का िह ा है
26. 113. Consider the following statements:
1. The Juvenile Justice Board is an institutional body constituted 26. 113. िन िल खत कथनों पर िवचार कीिजएः
under Section 4 of the Juvenile Justice Act, 2015. 1. िकशोर ाय बोड िकशोर ाय अिधिनयम, 2015 की धारा 4 के
तहत गिठत एक सं थागत िनकाय है ।
2. One or more than one Juvenile Justice Board(s) are
2. ेक रा के िलए रा ीय बाल अिधकार सं र ण आयोग
established by the National Commission for Protection of Child
(एनसीपीसीआर) ारा एक या एक से अिधक िकशोर ाय बोड
Rights (NCPCR) for each state.
थािपत िकए गए ह।
Which of the given above statements is/are correct?
ऊपर िदए गए कथनों म से कौन-सा/से सही है /ह?
(a) 1 only
(ए) केवल 1
(b) 2 only
(बी) केवल 2
(c) Both 1 and 2 (सी) 1 और 2 दोनों
(d) Neither 1 nor 2 (डी) न तो 1 और न ही 2
Ans: (a) उ र: (ए)
Explanation: ा ा:
• The Juvenile Justice Board is an institutional body constituted • िकशोर ाय बोड जेजे अिधिनयम, 2015 की धारा 4 के तहत गिठत
under Section 4 of the JJ Act, 2015. According to the division एक सं थागत िनकाय है । श यों के िवभाजन के अनुसार, आपरािधक
of powers, the subject of administration of criminal justice has ाय के शासन के िवषय को रा सू ची (सू ची II, एक या एक से
been included in the State List (List II, one or more than one अिधक) म शािमल िकया गया है ।
Juvenile Justice Board(s) are established by the State िकशोर ाय बोड ेक िजले के िलए रा सरकार ारा थािपत
Government for each district. The Board exercises its powers िकए जाते ह। बोड अपनी श यों का योग करता है और 'कानून के
and discharges functions relating to the ‘child in conflict with साथ सं घष म ब े' से सं बंिधत काय का िनवहन करता है जैसा िक इस
law’ as has been defined under Section 2(13) of this Act. अिधिनयम की धारा 2(13) के तहत प रभािषत िकया गया है ।

27.. In the context of any country, which one of the following 27.. िकसी भी दे श के सं दभ म, िन िल खत म से िकसे उसकी
would be considered as part of its social capital? सामािजक पूंजी का िह ा माना जाएगा?
(a) The proportion of literates in the population (ए) जनसं ा म सा रों का अनुपात
(b) The stock of its buildings, other infrastructure and machines (बी) इसकी इमारतों, अ बु िनयादी ढां चे और मशीनों का ॉक
(c) The size of population in the working age group (c) कामकाजी आयु वग म जनसं ा का आकार
(d) The level of mutual trust and harmony in the society (डी) समाज म आपसी िव ास और स ाव का र
Ans: (d) उ र: (डी)
Explanation: ा ा:
• Option A and D are examples of human capital and option B is िवक ए और डी मानव पूंजी के उदाहरण ह और िवक बी भौितक
physical capital. पूंजी है ।
• Straight from Wikipedia: “Social capital broadly refers to those • सीधे िविकपीिडया से : "सामािजक पूंजी ापक प से भावी
factors of effectively functioning social groups that include such सामािजक समूहों के उन कारकों को सं दिभत करती है िजनम
things as interpersonal relationships, a shared sense of identity, a पार रक सं बंध, पहचान की साझा भावना, साझा समझ, साझा
shared understanding, shared norms, shared values, trust, मानदं ड, साझा मू , िव ास, सहयोग और पार रकता जैसी चीज
cooperation, and reciprocity.” Option D is the correct शािमल ह।" िवक D सही है
answer. जवाब।
28. Consider the following statements: 28. िन िल खत कथनों पर िवचार कीिजएः
1. Whenever forest land is diverted for non-forest purposes, it is 1. जब भी वन भू िम को गैर-वन उ े ों के िलए बदला जाता है , तो वन
mandatory under the Forest Rights Act (FRA), 2006 that an अिधकार अिधिनयम (एफआरए), 2006 के तहत यह अिनवाय है िक
equivalent area of non-forest land has to be taken up for
गै र-वन भू िम के बराबर े को ितपूरक वनीकरण के िलए िलया
compensatory afforestation.
जाना चािहए।
2. As per the rules, 90% of the Compensatory Afforestation
Fund money is to be given to the states while 10% is to be 2. िनयमों के अनुसार, ितपूरक वनीकरण कोष का 90% पैसा रा ों
retained by the Centre. को िदया जाना है जबिक 10% क ारा रखा जाना है ।
3. As per recent notification by GOI, states which have a forest 3. भारत सरकार की हािलया अिधसू चना के अनुसार, िजन रा ों के
land of more than 75% of their geographical area need not भौगोिलक े का 75% से अिधक वन भू िम है , उ ितपूरक
provide non-forest land for compensatory afforestation. वनीकरण के िलए गै र-वन भू िम दान करने की आव कता नहीं है ।
Which of the given above statements is/are correct?
ऊपर िदए गए कथनों म से कौन-सा/से सही है /ह?
(a) 1 and 2 only
(ए) केवल 1 और 2
(b) 2 only
(बी) केवल 2
(c) 1 and 3 only
(सी) केवल 1 और 3
(d) 2 and 3 only
(डी) केवल 2 और 3
Ans: (d)
Explanation: उ र: (डी)
• S3: the Ministry of Environment notified that states which have ा ा:
a forest land of more than 75% of their geographical area need • S3: पयावरण मं ालय ने अिधसू िचत िकया िक िजन रा ों के पास
not provide non-forest land for compensatory afforestation. उनके भौगोिलक े के 75% से अिधक वन भू िम है , उ ितपूरक
Instead, land can be taken up in states with lesser forest cover.
वनीकरण के िलए गै र-वन भू िम दान करने की आव कता नहीं है ।
Further, it was also notified that the minimum area of
इसके बजाय, कम वन आ ादन वाले रा ों म भूिम ली जा सकती है ।
compensatory land should be five hectares if the land is not
contiguous to a forest. इसके अलावा, यह भी अिधसू िचत िकया गया था िक ूनतम े
• S1: According to the Forest (Conservation) Act, 1980, each यिद भूिम जंगल से सटी ई नहीं है तो ितपूरक भू िम पां च हे े यर
time forest land is diverted, the project proponent has to pay the होनी चािहए।
state to undertake plantation and for the ecosystem services lost • S1: वन (सं र ण) अिधिनयम, 1980 के अनुसार, हर बार जब वन
due to diverting forest land, called Net Present Value (NPV). भू िम का उपयोग िकया जाता है, तो प रयोजना ावक को रा को
Whenever forest land is diverted for non-forest purposes, it is वृ ारोपण करने के िलए भु गतान करना पड़ता है और वन भू िम के
mandatory under the Forest (Conservation) Act, 1980 that an िवचलन के कारण खोई ई पा र थितकी तं से वाओं के िलए, िजसे
equivalent area of non-forest land has to be taken up for शु वतमान मू (NPV) कहा जाता है ). जब भी वन भू िम को गै र-वन
compensatory afforestation. उ े ों के िलए डायवट िकया जाता है, यह है
• S2: As per the rules, 90% of the Compensatory Afforestation वन (सं र ण) अिधिनयम, 1980 के तहत अिनवाय है िक ितपूरक
Fund money is to be given to the states while 10% is to be वनीकरण के िलए गै र-वन भू िम के बराबर े को िलया जाना है ।
retained by the Centre. • S2: िनयमों के अनुसार, ितपूरक वनीकरण कोष का 90% पैसा
रा ों को िदया जाना है जबिक 10% क ारा रखा जाना है ।
29. 134. ‘Namsai Declaration’ was in news recently, is related to
(a) good governance 29. 134. हाल ही म खबरों म रहा 'नामसाई घोषणा' िकससे सं बंिधत है
(b) interlinking of rivers (ए) सु शासन
(c) disaster management (b) निदयों को आपस म जोड़ना
(d) settling border dispute (सी) आपदा बं धन
Ans: (d) (d) सीमा िववाद सु लझाना
Explanation: Namsai Declaration: उ र: (डी)
• This declaration is signed between the states of Assam and ा ा: नमसाई घोषणा:
Arunachal Pradesh on July 15, 2022, at Namsai, Arunachal • अ णाचल दे श ारा थानीय आयोग के सम रखे गए 123 गां वों
Pradesh to minimize the border dispute between the two के सं बंध म दोनों रा ों के बीच सीमा िववाद को कम करने के िलए
states in respect of 123 villages placed before the Local अ णाचल दे श के नमसाई म 15 जुलाई, 2022 को असम और
Commission by Arunachal Pradesh. अ णाचल दे श रा ों के बीच इस घोषणा पर ह ा र िकए गए ह।
• According to the declaration, all border issues between Assam • घोषणा के अनुसार, असम और अ णाचल दे श के बीच सभी सीमा
and Arunachal Pradesh will be confined to those raised before मु े 2007 म थानीय आयोग के सम उठाए गए मु ों तक ही सीिमत
the local commission in 2007. रहगे ।
30. Consider the following statements: 30. िन िल खत कथनों पर िवचार कीिजएः
1. Constitution of India does not define the word bail but only 1. भारत का संिवधान जमानत श को प रभािषत नहीं करता है ब
categories offences as ‘bailable’ and ‘non-bailable’. केवल अपराधों को 'जमानती' और 'गै र-जमानती' के प म वग कृत
2. Constitution of India empowers magistrates to grant bail for करता है ।
bailable offences as a matter of right. 2. भारत का सं िवधान मिज े टों को अिधकार के प म जमानती
Which of the statements given above is/are correct? अपराधों के िलए जमानत दे ने का अिधकार दे ता है ।
(a) 1 only ऊपर िदए गए कथनों म से कौन सा/से सही है /ह?
(b) 2 only (ए) केवल 1
(c) Both 1 and 2 (बी) केवल 2
(सी) 1 और 2 दोनों
(d) Neither 1 nor 2
(डी) न तो 1 और न ही 2
Ans: (d)
उ र: (डी)
Explanation:
ा ा:
• The CrPC does not define the word bail but only categories
offences under the Indian Penal Code as ‘bailable’ and ‘non- • सीआरपीसी जमानत श को प रभािषत नहीं करता है ब
bailable’. भारतीय दं ड सं िहता के तहत केवल 'जमानती' और 'गै र-जमानती' के
• The CrPC empowers magistrates to grant bail for bailable प म ेणीब करता है ।
offences as a matter of right. • सीआरपीसी मिज े टों को अिधकार के प म जमानती अपराधों के
• This would involve release on furnishing a bail bond, without िलए जमानत दे ने का अिधकार दे ता है ।
or without security. • इसम सु र ा के िबना या िबना जमानत बां ड ु त करने पर रहाई
• In the case of Non-bailable offences, a magistrate would शािमल होगी।
determine if the accused is fit to be released on bail. • गै र-जमानती अपराधों के मामले म, एक मिज े ट िनधा रत करे गा िक
• Non-bailable offences are cognisable, which enables the police अिभयु जमानत पर रहा होने के यो है या नहीं।
officer to arrest without a warrant. • गै र-जमानती अपराध सं ेय ह, जो पुिलस अिधकारी को िबना वारं ट
के िगर ार करने म स म बनाता है ।
31. Consider the following statements regarding the Office of
Vice President of India. 31. भारत के उपरा पित के कायालय के सं बंध म िन िल खत कथनों
1. The Vice President takes over the office of the President when पर िवचार कर।
there is a vacancy. 1. उपरा पित रा पित का पद र होने पर हण करता है ।
2. The Vice President is removed from his office by a resolution 2. उपरा पित को सं सद के दोनों सदनों ारा िवशेष ब मत से पा रत
of both the houses of the parliament by a special majority. ाव ारा उसके पद से हटाया जाता है ।
Which of the above statements is/are correct? उपरो कथनों म से कौन-सा/से सही है /ह?
(a) 1 only (ए) केवल 1
(b) 2 only (बी) केवल 2
(c) Both 1 and 2 (सी) 1 और 2 दोनों
(d) Neither 1 nor 2 (डी) न तो 1 और न ही 2
Ans: (a) उ र: (ए)
Explanation: ा ा:
• The Vice President is elected for five years. His election • उपरा पित का चुनाव पां च वष के िलए िकया जाता है । उनकी चुनाव
method is similar to that of the President, the only difference is प ित रा पित के समान है, केवल अंतर यह है िक रा िवधानसभाओं
that members of State legislatures are not part of the electoral के सद िनवाचक मंडल का िह ा नही ं होते ह।
college. • उपरा पित केवल एक नए रा पित के होने तक रा पित के पम
• The Vice President acts as the President only until a new काय करता है
President is चुने ए। फख ीन अली अहमद की मृ ु पर बी डी ज ी ने रा पित
elected. B. D. Jatti acted as President on the death of Fakhruddin के प म काय िकया जब तक िक एक नया रा पित नहीं चुना गया।
Ali Ahmed until a new President was elected. • उपरा पित का महािभयोग रा पित के महािभयोग से िभ होता है ।
• The impeachment of Vice President is different from that of the बाद के मामले म, दोनों सदनों को िवशेष ब मत से ाव पा रत
President. In the latter case, both houses must pass the resolution करना चािहए।
by a special majority. • लेिकन वीपी को रा सभा के एक भावी ब मत से पा रत एक
• But the VP may be removed from his office by a resolution ाव ारा अपने कायालय से हटाया जा सकता है और लोकसभा
of the Rajya Sabha passed by an effective majority and ारा साधारण ब मत से सहमित की जा सकती है ।
agreed to by the Lok Sabha with simple majority.
32. 145. उप-रा पित के प म चुनाव के िलए पा होने के िलए, एक
32. 145. To be eligible for election as Vice-President, a person
should fulfil which of the following conditions? िन िल खत म से िकन शत को पू रा करना चािहए?
1. He should not be less than 30 years of age. 1. उसकी आयु 30 वष से कम नहीं होनी चािहए।
2. He should be qualified for election as a member of the Rajya 2. वह रा सभा के सद के प म चुनाव के िलए यो होना चािहए।
Sabha. 3. वह क सरकार या िकसी रा सरकार या िकसी थानीय ािधकरण
3. He should not hold any office of profit under the Union या िकसी अ सावजिनक ािधकरण के अधीन िकसी लाभ के पद पर न
government or any state government or any local authority or हो।
any other public authority. सही उ र कूट का चयन कर:
Select the correct answer code:
क) केवल 1 और 2
a) 1 and 2 only
b) 2 only बी) केवल 2
c) 2 and 3 only c) केवल 2 और 3
d) 1, 2 and 3 डी) 1, 2 और 3
Ans: (c) उ र: (सी)
Explanation: ा ा:
• To be eligible for election as Vice-President, a person should • उपरा पित के प म चुनाव के िलए यो होने के िलए, एक को
fulfil the following qualifications: िन िल खत यो ताएं पू री करनी चािहए:
o He should be a citizen of India. o वह भारत का नाग रक होना चािहए।
o He should have completed 35 years of age. o उसे 35 वष की आयु पू री कर लेनी चािहए थी।
o He should be qualified for election as a member of the
o वह रा सभा के सद के प म चुनाव के िलए यो होना चािहए।
Rajya Sabha.
o He should not hold any office of profit under the Union 0 वह क सरकार या िकसी रा सरकार या िकसी थानीय ािधकरण या
government or any state government or any local authority िकसी अ सावजिनक ािधकरण के अधीन िकसी लाभ के पद पर न हो।
or any other public authority.
33. भारत म 'अ सं क' (या 'अ सं क') के संदभ म, िन िल खत म
33. With reference to ‘Minorities’ (or ‘Minority’) in India, which से कौन सा/से कथन सही है /ह?
of the following statements is/are correct? 1. सभी अ सं कों को अपनी पसंद के िश ण सं थानों की थापना और
1. All minorities have the right to establish and administer शासन का अिधकार है ।
educational institutions of their choice. 2. अनु े द 29 के तहत सुर ा केवल अ सं कों तक ही सीिमत है और
2. The protection under Article 29 is confined only to minorities नाग रकों के िकसी भी वग तक िव ा रत नहीं है ।
and does not extend to any section of citizens. नीचे िदए गए कूट का योग कर सही उ र चुिनए:
Select the correct answer using the codes given below: (ए) केवल 1
(a) 1 only (बी) केवल 2
(b) 2 only
(सी) 1 और 2 दोनों
(c) Both 1 and 2
(d) Neither 1 nor 2 (डी) न तो 1 और न ही 2
Ans: (a) उ र: (ए)
Explanation: ा ा:
• Article 29: • अनु े द 29:
o It provides that any section of the citizens residing in any part 0 यह ावधान करता है िक भारत के िकसी भी िह े म रहने वाले
of India having a distinct language, script or culture of its नाग रकों के िकसी भी वग की अपनी एक अलग भाषा, िलिप या सं ृ ित है ,
own, shall have the right to conserve the same. उसे इसके संर ण का अिधकार होगा।
o It grants protection to both religious minorities as well as o यह धािमक अ सं कों के साथ-साथ भाषाई अ सं कों दोनों को
linguistic minorities. सुर ा दान करता है ।
o However, the SC held that the scope of this article is not
हालां िक, सु ीम कोट ने माना िक इस अनु े द का दायरा केवल
necessarily restricted to minorities only, as the use of the word
अ सं कों तक ही सीिमत नहीं है , ोंिक अनु े द म 'नाग रकों के वग'
‘section of citizens’ in the Article includes minorities as well as
the majority. श के उपयोग म अ सं कों के साथ-साथ ब सं क भी शािमल ह।
• Article 30: • अनु े द 30:
o S1: All minorities shall have the right to establish and 0 S1: सभी अ सं कों को अपनी पसंद के िश ण सं थानों की थापना
administer educational institutions of their choice. और शासन का अिधकार होगा।
o S2: The protection under Article 30 is confined only to o S2: अनु े द 30 के तहत सुर ा केवल अ सं कों (धािमक या
minorities (religious or linguistic) and does not extend to any भाषाई) तक ही सीिमत है और यह नाग रकों के िकसी भी वग तक
section of citizens (as under Article 29). िव ा रत नहीं है (अनु े द 29 के तहत)।

34. 152. With reference to the Special Officer for Linguistic 34. 152. भाषाई अ सं कों के िलए िवशेष अिधकारी के संदभ म,
Minorities, consider the following statements: िन िल खत कथनों पर िवचार कर:
1. Special Officer for Linguistic Minorities has been inserted to 1. भाषाई अ सं कों के िलए िवशेष अिधकारी को 42व संिवधान
the constitution by the 42nd Constitutional Amendment Act.
संशोधन अिधिनयम ारा संिवधान म स िलत िकया गया है ।
2. It would be the duty of the Special Officer to investigate all
2. संिवधान के तहत भाषाई अ सं कों के िलए दान िकए गए सुर ा
matters relating to the safeguards provided for linguistic
उपायों से संबंिधत सभी मामलों की जां च करना िवशेष अिधकारी का
minorities under the Constitution.
कत होगा।
Which of the statements given above is/are correct? ऊपर िदए गए कथनों म से कौन सा/से सही है /ह?
(a) 1 only (ए) केवल 1
(b) 2 only (बी) केवल 2
(c) Both 1 and 2 (सी) 1 और 2 दोनों
(d) Neither 1 nor 2 (डी) न तो 1 और न ही 2
Ans: (b) उ र: (बी)
Explanation: ा ा:
• Originally, the Constitution of India did not make any provision • मूल प से, भारत के संिवधान म भाषाई अ सं कों के िलए िवशेष
with respect to the Special Officer for Linguistic Minorities.
अिधकारी के संबंध म कोई ावधान नहीं था। बाद म, रा पु नगठन
Later, the States Reorganization Commission (1953-55) made a
recommendation in this regard. Accordingly, the 7th आयोग (1953-55) ने इस संबंध म एक िसफा रश की। तदनुसार, 1956 के
Constitutional Amendment Act of 1956 inserted a new Article 7व संवैधािनक संशोधन अिधिनयम ने संिवधान के भाग XVII म एक नया
350-B in Part XVII of the Constitution. This article contains the अनु े द 350-बी डाला। इस अनु े द म िन िल खत ावधान ह: भाषाई
following provisions: There should be a Special Officer for अ सं कों के िलए एक िवशेष अिधकारी होना चािहए। उ भारत के
Linguistic Minorities. He is to be appointed by the President of रा पित ारा िनयु िकया जाना है ।
India. • संिवधान के तहत भाषाई अ सं कों के िलए दान िकए गए सुर ा
• It would be the duty of the Special Officer to investigate all उपायों से संबंिधत सभी मामलों की जां च करना िवशेष अिधकारी का
matters relating to the safeguards provided for linguistic कत होगा। वह ऐसे अंतरालों पर उन मामलों पर रा पित को रपोट
minorities under the Constitution. Hewould report to the करे गा
President upon those matters at such intervals as the
रा पित िनदिशत कर सकते ह। रा पित को ऐसी सभी रपोट संसद के
President may direct. The President should place all such reports
ेक सदन के सम रखनी चािहए और संबंिधत रा ों की सरकारों को
before each House of Parliament and send to the governments of
the states concerned. भे जनी चािहए।

35. In India, legal service authorities provide free legal services 35. भारत म, कानूनी सेवा ािधकरण िन िल खत म से िकस कार के
to which of the following type of citizens? नाग रकों को मु कानूनी सेवाएं दान करते ह?
1. Person with an annual income of less than Rs 1,00,000 1. 1,00,000 पये से कम वािषक आय वाले
2. Transgender with an annual income of less than Rs2,00,000 2. 2,00,000 पये से कम वािषक आय वाले टां सजडर
3. Member of other backward classes(OBC) with an annual 3. अ िपछड़ा वग (ओबीसी) के सद िजनकी वािषक आय 3,00,000
income of less than Rs 3,00,000 पये से कम है
4. All senior citizens 4. सभी व र नाग रक
Select the correct answer using the given code below- नीचे िदए गए कूट का योग कर सही उ र चुिनए-
(a) 1 and 2 only (ए) केवल 1 और 2
(b) 3 and 4 only (बी) केवल 3 और 4
(c) 2 and 3 only (सी) केवल 2 और 3
(d) 1 and 4 only (डी) केवल 1 और 4
Ans: (a)
उ र: (ए)
Explanation:
• S1 and S3: The sections of the society as enlisted under Section ा ा:
12 of the Legal Services Authorities Act are entitled for free • S1 और S3: कानूनी सेवा ािधकरण अिधिनयम की धारा 12 के तहत
legal services, they are : सूचीब समाज के वग मु कानू नी सेवाओं के हकदार ह, वे ह:
o A member of a Scheduled Caste or Scheduled Tribe; o अनुसूिचत जाित या अनुसूिचत जनजाित का सद ;
o A victim of trafficking in human beings or begar as referred to 0 संिवधान के अनु े द 23 म संदिभत मानव त री या बेगार का िशकार;
in Article 23 of the Constitution; o एक मिहला या एक ब ा;
o A woman or a child; o मानिसक प से बीमार या अ था िवकलां ग ;
o A mentally ill or otherwise disabled person; 0 ापक आपदा, जातीय िहं सा, जातीय अ ाचार, बाढ़, सूखा, भू कंप या
o A person under circumstances of undeserved want such as औ ोिगक आपदा का िशकार होने जैसी अपा अभाव की प र थितयों म
being a victim of a mass disaster, ethnic violence, caste atrocity,
एक ; या
flood, drought, earthquake or industrial disaster; or
ओ एक औ ोिगक कमकार; या
o An industrial workman; or
o In custody, including custody in a protective home within the o अनैितक ापार (रोकथाम) अिधिनयम, 1956 (1956 का 104) की धारा
meaning of clause (g) of Section 2 of the Immoral Traffic 2 के खंड (जी) के अथ के भीतर एक सुर ा क घर म िहरासत सिहत
(Prevention) Act, 1956(104 of 1956); or in a juvenile home िहरासत म; या िकशोर ाय अिधिनयम, 1986 (1986 का 53) की धारा 2
within the meaning of clause(j) of Section 2 of the Juvenile के खंड (जे) के अथ म एक िकशोर गृह म; या मानिसक ा अिधिनयम,
Justice Act, 1986 (53 of 1986); or in a psychiatric hospital or 1987 (1987 का 14) की धारा 2 के खंड (जी) के अथ के भीतर एक
psychiatric nursing home within the meaning of clause (g) of मनोरोग अ ताल या मनोरोग निसग होम म; या 0 िन िल खत अनुसूची म
Section 2 of the Mental Health Act, 1987(14 of 1987);or o a उ खत रािश से कम वािषक आय ा करने वाला (या कोई
person in receipt of annual income less than the amount अ उ रािश जो रा सरकार ारा िनधा रत की जा सकती है ), यिद
mentioned in the following schedule (or any other higher amount
मामला सव ायालय के अलावा िकसी अ ायालय के सम है ,
as may be prescribed by the State Government), if the case is
और 5 पये से कम है
before a Court other than the Supreme Court, and less than Rs 5
Lakh, if the case is before the Supreme Court. लाख, अगर मामला सु ीम कोट के सम है ।
• The Income Ceiling Limit prescribed u/S 12(h) of the Act for • िविभ रा ों म मु कानूनी सेवाओं का लाभ उठाने के िलए अिधिनयम
availing free legal services in different States has been stated की धारा 12(एच) के तहत िनधा रत आय की उ तम सीमा यहां बताई गई
here https://nalsa.gov.in/faqs है https://nalsa.gov.in/faqs
• S2: Limit for Transgender – Rs. 2,00,000 see
• S2: टां सजडर के िलए सीमा - . 2,00,000 https://nalsa.gov.in/faqs
https://nalsa.gov.in/faqs
• S4: Senior citizens’ eligibility for free legal aid depends on the दे ख
Rules framed by the respective State Governments in this regard. • S4: मु कानूनी सहायता के िलए व र नाग रकों की पा ता संबंिधत
• In Delhi for example, senior citizens are eligible for free legal रा सरकारों ारा इस संबंध म बनाए गए िनयमों पर िनभर करती है ।
aid subject to prescribed ceiling of annual income. Any • उदाहरण के िलए, िद ी म व र नाग रक वािषक आय की िनधा रत
individual above the age of 60 can apply for free legal aid/ सीमा के अधीन मु कानूनी सहायता के पा ह। 60 वष से अिधक आयु
services. का कोई भी िनःशु कानूनी सहायता के िलए आवेदन कर सकता
है / सेवाएं ।
36. Consider the following statements regarding National Legal
Services Authority (NALSA). 36. रा ीय कानूनी सेवा ािधकरण (एनएएलएसए) के संबंध म िन िल खत
1. The National Legal Services Authority (NALSA) has been कथनों पर िवचार कर।
constituted under the Legal Services Authorities Act, 1987 to 1. रा ीय िविधक सेवा ािधकरण (NALSA) का गठन कानूनी सेवा
provide free Legal Services to the weaker sections of the society. ािधकरण अिधिनयम, 1987 के तहत समाज के कमजोर वग को मु
2. The President of India is the Patron-in-Chief of the Authority. कानूनी सेवाएं दान करने के िलए िकया गया है ।
Which of the above statements is/are correct?
2. भारत के रा पित ािधकरण के संर क-इन-चीफ ह।
(a) 1 only
(b) 2 only उपरो कथनों म से कौन-सा/से सही है /ह?
(c) Both 1 and 2 (ए) केवल 1
(d) Neither 1 nor 2 (बी) केवल 2
Ans: (a) (सी) 1 और 2 दोनों
Explanation: (डी) न तो 1 और न ही 2
• The National Legal Services Authority (NALSA) has been उ र: (ए)
constituted under the Legal Services Authorities Act, 1987 to ा ा:
provide free Legal Services to the weaker sections of the society • रा ीय िविधक सेवा ािधकरण (NALSA) का गठन कानूनी सेवा
and to organize Lok Adalats for amicable settlement of disputes. ािधकरण अिधिनयम, 1987 के तहत समाज के कमजोर वग को मु
• The Chief Justice of India is patron-in-chief of NALSA while
कानूनी सेवाएं दान करने और िववादों के सौहादपू ण समाधान के िलए
second senior most judge of Supreme Court of India is the
Executive-Chairman. लोक अदालतों का आयोजन करने के िलए िकया गया है ।
• In every State, State Legal Services Authority has been • भारत के मु ायाधीश नालसा के संर क-इन-चीफ ह, जबिक भारत
constituted to give effect to the policies and directions of the के सव ायालय के दू सरे व र तम ायाधीश कायकारी-अ ह।
NALSA and to give free legal services to the people and conduct • नालसा की नीितयों और िनदशों को लागू करने और लोगों को मु
Lok Adalats in the State. कानूनी सेवाएं दे ने और रा म लोक अदालत आयोिजत करने के िलए हर
रा म रा िविधक सेवा ािधकरण का गठन िकया गया है ।
37. Which of the following is/are the demerits of the
Parliamentary System? 37. िन िल खत म से कौन-सा/से संसदीय णाली के दोष ह/ह?
1. Government by Amateurs 1. शौकीनों ारा सरकार
2. Not conductive to administrative efficiency 2. शासिनक द ता के अनुकूल नहीं
3. Separation of Powers 3. श यों का पृ थ रण
Select the correct answer code: सही उ र कूट का चयन कर:
(a) 1 only (ए) केवल 1
(b) 2 and 3 only
(बी) केवल 2 और 3
(c) 1 and 2 only
(d) 1, 2 and 3 (सी) केवल 1 और 2
Ans: (c) (डी) 1, 2 और 3
Explanation: उ र: (सी)
• Government by Amateurs: the parliamentary system is not ा ा:
conductive to administrative efficiency as the ministers are not • शौकीनों ारा सरकार: संसदीय णाली शासिनक द ता के अनुकूल
experts in their fields. The Prime Minister has a limited choice in नहीं है ोंिक मं ी अपने े के िवशेष नहीं होते ह। मंि यों के चयन म
the selection of ministers; his choice is restricted to the members धानमं ी के पास सीिमत िवक होते ह; उनकी पसंद केवल संसद के
of Parliament alone and does not extend to external talent. सद ों तक ही सीिमत है और बाहरी ितभा तक नहीं है ।
• In the parliamentary system, the legislature and the executive • संसदीय णाली म, िवधाियका और कायपािलका एक साथ और
are together and inseparable. The cabinet acts as the leader of
अिवभा ह। कैिबनेट िवधाियका के साथ-साथ कायकारी के नेता के प
legislature as well as the executive.
म काय करता है ।
38. Consider the following statements about the Competition
Commission of India (CCI): 38. भारतीय ित धा आयोग (CCI) के बारे म िन िल खत कथनों पर
िवचार कर:
1. It acts as the competition regulator in India. 1. यह भारत म ित धा िनयामक के प म काय करता है ।
2. It is working under the Ministry of Commerce and Industry. 2. यह वािण एवं उ ोग मं ालय के अधीन कायरत है । ऊपर िदए गए
Which of the statements given above is/are correct? कथनों म से कौन सा/से सही है /ह?
(a) 1 only (ए) केवल 1
(b) 2 only (बी) केवल 2
(c) Both 1 and 2 (सी) 1 और 2 दोनों
(d) Neither 1 nor 2 (डी) न तो 1 और न ही 2
Ans: (a) उ र: (ए)
Explanation: ा ा:
• The Competition Commission of India (CCI) is a statutory • भारतीय ित धा आयोग (सीसीआई) कॉप रे ट मामलों के मं ालय के
and quasi-judicial body working under the Ministry of
तहत काम करने वाला एक वैधािनक और अध- ाियक िनकाय है । यह
Corporate Affairs. It was established under the Competition
अिधिनयम के शासन, काया यन और वतन के िलए ित धा
Act, 2002 for the administration, implementation and
enforcement of the Act, and was duly constituted in March 2009. अिधिनयम, 2002 के तहत थािपत िकया गया था, और माच 2009 म
िविधवत गिठत िकया गया था।
39. Consider the following statements:
1. The guidelines for renunciation of Indian citizenship is issued 39. िन िल खत कथनों पर िवचार कीिजएः
by the Union Ministry of Home Affairs (MHA). 1. भारतीय नाग रकता के ाग के िलए िदशािनदश क ीय गृह मं ालय
2. Every minor child of the person who ceases to be a citizen of (एमएचए) ारा जारी िकए जाते ह।
India shall also cease to be a citizen of India. 2. भारत का नाग रक न रहने वाले की ेक अवय संतान भी
Which of the above statements is/are correct? भारत की नाग रक नहीं रहे गी।
(a) 1 only उपरो कथनों म से कौन-सा/से सही है /ह?
(b) 2 only (ए) केवल 1
(c) Both 1 and 2 (बी) केवल 2
(d) Neither 1 nor 2 (सी) 1 और 2 दोनों
Ans: (c) (डी) न तो 1 और न ही 2
Explanation: उ र: (सी)
• The citizenship act, of 1955 prescribes three ways of losing ा ा:
citizenship:
• 1955 का नाग रकता अिधिनयम, नाग रकता खोने के तीन तरीके
o By renunciation: Any citizen of India of full age and capacity
can make a declaration renouncing Indian citizenship. Such िनधा रत करता है :
declaration may not be accepted during the war. Even the minor 0 ाग ारा: पू ण आयु और मता वाला भारत का कोई भी नाग रक
children of the person who renounces citizenship stand to lose भारतीय नाग रकता ागने की घोषणा कर सकता है । यु के दौरान ऐसी
their Indian citizenship. However, when their children attain the घोषणा को ीकार नहीं िकया जा सकता है । यहां तक िक नाग रकता
age of eighteen, they may resume Indian citizenship छोड़ने वाले के अवय ब े भी अपनी भारतीय नाग रकता खो
o By termination: If a citizen of India voluntarily acquires the दे ते ह। हालाँ िक, जब उनके ब े अठारह वष की आयु ा करते ह, तो वे
citizenship of another country, then he loses the citizenship of भारतीय नाग रकता िफर से शु कर सकते ह
India o समा ारा: यिद भारत का कोई नाग रक े ा से िकसी अ दे श
o By deprivation: Compulsory termination of Indian citizenship की नाग रकता ा करता है , तो वह भारत की नाग रकता खो दे ता है
by the Central government, if the person obtained the citizenship
o वंचन ारा: क सरकार ारा भारतीय नाग रकता की अिनवाय समा ,
by fraud or shown disloyalty to the Constitution of India or
यिद ने धोखे से नाग रकता ा की है या भारत के संिवधान के ित
imprisoned for a term of two years (in the last 5 years after
naturalization) or citizen has been ordinarily resident out of India अिन ा िदखाई है या दो साल की अविध के िलए कारावास ( ाकृितककरण
for a period of 7 years के बाद िपछले 5 वष म) या नाग रक ने 7 वष की अविध के िलए सामा
• The guidelines said that when a person ceases to be a citizen of प से भारत से बाहर िनवासी रहा हो
India under Section 8(1) of Citizenship Act, 1955, “every minor • िदशािनदशों म कहा गया है िक जब कोई नाग रकता अिधिनयम,
child of that person shall thereupon ceases to be a citizen of 1955 की धारा 8(1) के तहत भारत का नाग रक नहीं रहता है , तो "उस
India”. The minor child may, however, within one year of का ेक नाबािलग ब ा भारत का नाग रक नहीं रहे गा"।
attaining full age apply to resume Indian citizenship. The नाबािलग ब ा, हालां िक, पू ण आयु ा करने के एक वष के भीतर
guidelines are not clear if minors would also lose citizenship if भारतीय नाग रकता को िफर से शु करने के िलए आवेदन कर सकता है ।
only one of the parents gives up her/his Indian citizenship.
िदशा-िनदश नहीं ह िक ा नाबािलग भी नाग रकता खो दगे यिद
माता-िपता म से केवल एक ही अपनी भारतीय नाग रकता छोड़ दे ता है ।
40. Consider the following statements regarding National
Commission for Backward Classes (NCBC).
1. National Commission for Backward Classes is a non- 40. रा ीय िपछड़ा वग आयोग (एनसीबीसी) के संबंध म िन िल खत कथनों
constitutional body under the Ministry of Social Justice and पर िवचार कर।
Empowerment. 1. रा ीय िपछड़ा वग आयोग सामािजक ाय और अिधका रता मं ालय के
2. The commission was the outcome of Indra Sawhney & Ors. तहत एक गैर-संवैधािनक िनकाय है ।
Vs. Union of India. 2. आयोग इं ा साहनी और अ का प रणाम था। बनाम भारत संघ।
3. The commission have the same powers as a Civil Court. 3. आयोग के पास िसिवल कोट के समान श यां ह। उपरो कथनों म
Which of the above statements is/are correct? से कौन-सा/से सही है /ह?
(a) 1 and 2 only (ए) केवल 1 और 2
(b) 2 and 3 only (बी) केवल 2 और 3
(c) 1 and 3 only (सी) केवल 1 और 3
(d) 1, 2 and 3
(डी) 1, 2 और 3
Ans: (b)
Explanation: उ र: (बी)
• National Commission for Backward Classes is a constitutional ा ा:
body (123rd constitutional amendment bill 2017 and 102nd रा ीय िपछड़ा वग आयोग 14 अग 1993 को थािपत सामािजक ाय
amendment 2018 in constitution to make it constitutional
और अिधका रता मं ालय के तहत एक संवैधािनक िनकाय (123वां
body) (Article 338B of the Indian Constitution) under Ministry
of Social Justice and Empowerment established on 14 August संवैधािनक संशोधन िवधेयक 2017 और संिवधान म 102वां संशोधन 2018
1993. इसे संवैधािनक िनकाय बनाने के िलए) (भारतीय संिवधान का अनु े द
• It was constituted pursuant to the provisions of the National 338B) है ।
Commission for Backward Classes Act, 1993. The commission
was the outcome of Indra Sawhney & Ors. Vs. Union of India. • इसका गठन रा ीय िपछड़ा वग आयोग अिधिनयम, 1993 के ावधानों के
• The National Commission for Backward Classes, National अनुसार िकया गया था। यह आयोग इं ा साहनी और अ का प रणाम था।
Commission for Scheduled Castes as well as National बनाम भारत संघ।
Commission for Scheduled Tribes have the same powers as a • रा ीय िपछड़ा वग आयोग, रा ीय अनुसूिचत जाित आयोग और रा ीय
Civil Court. अनुसूिचत जनजाित आयोग के पास िसिवल कोट के समान अिधकार ह।

41. The 102nd Constitutional Amendment Act, sometimes seen 41. कभी-कभी समाचारों म दे खा जाने वाला 102वां संिवधान संशोधन
in news is related to
अिधिनयम संबंिधत है
(a) 10% reservation for Economically weaker sections
(b) Constitutional status to National Commission for Backward (ए) आिथक प से कमजोर वग के िलए 10% आर ण
Classes (बी) रा ीय िपछड़ा वग आयोग को संवैधािनक दजा
(c) Extend reservation for Scheduled Castes (SCs) and (सी) लोकसभा और िवधायी िनकायों के िलए अनुसूिचत जाित (एससी) और
Scheduled Tribes (STs) to Lok Sabha and legislative bodies अनुसूिचत जनजाित (एसटी) के िलए आर ण का िव ार
(d) Abolition of Legislative Councils in certain States (d) कुछ रा ों म िवधान प रषदों का उ ू लन
Ans: (b)
Explanation: उ र: (बी)
• The One Hundred and Second Amendment of the Constitution ा ा:
of India, officially known as the Constitution (One Hundred and • भारत के संिवधान का एक सौ दू सरा संशोधन, िजसे आिधका रक तौर पर
Second Amendment) Act, 2018, granted constitutional status to
संिवधान (एक सौ दू सरा संशोधन) अिधिनयम, 2018 के प म जाना जाता
the National Commission for Backward Classes (NCBC).
है , ने रा ीय िपछड़ा वग आयोग (NCBC) को संवैधािनक दजा िदया।
42. Which of the following statements is / are correct regarding
the Medical Termination of Pregnancy (MTP) Amendment Act, 42. मेिडकल टिमनेशन ऑफ े गनसी (एमटीपी) संशोधन अिधिनयम,
2021?
2021 के संबंध म िन िल खत म से कौन सा/से कथन सही है /ह?
1. It extends the upper limit for medical termination of
pregnancy to 24 weeks from 20 weeks, for certain categories of 1. यह मिहलाओं की कुछ ेिणयों के िलए िचिक ीय गभपात की ऊपरी
women. सीमा को 20 स ाह से बढ़ाकर 24 स ाह कर दे ता है ।
2. The opinion of a registered medical practitioner, eligible as 2. 20 स ाह तक की िविभ गभकालीन आयु म गभ समापन के िलए
per the rules is required for the termination of pregnancy at िनयमानुसार पा पं जीकृत िचिक क की राय आव क है ।
different gestation ages up to 20 weeks.
3. बीस स ाह से अिधक की गभ अविध के िचिक ीय समापन के िलए
3. The opinion for medical termination of pregnancy beyond
twenty weeks gestation period will be given by a medical board राय संबंिधत रा सरकार या संघ रा े शासन ारा अनुमोिदत
duly constituted by the respective state government or UT सुिवधाओं पर िविधवत गिठत मेिडकल बोड ारा दी जाएगी।
administration at approved facilities. नीचे िदए गए कूट का योग कर सही उ र चुिनए।
Select the correct answer using the code given below.
(ए) केवल 1 और 2
(a) 1 and 2 only
(b) 2 only (बी) केवल 2
(c) 3 only (सी) केवल 3
(d) 1, 2 and 3 (डी) 1, 2 और 3
Ans: (a) उ र: (ए)
Explanation: The Medical Termination of Pregnancy
ा ा: गभाव था का िचिक कीय समापन (संशोधन) अिधिनयम, 2021
(Amendment) Act, 2021
• The Act amends the 1971 MTP law which regulates the • अिधिनयम 1971 के एमटीपी कानून म संशोधन करता है जो उन शत
conditions under which medical termination of pregnancy is to को िनयंि त करता है िजनके तहत गभाव था का िचिक ीय समापन िकया
be pursued. The amended MTP Act extends the upper limit जाना है । संशोिधत एमटीपी अिधिनयम गभाव था के िचिक ीय समापन
for medical termination of pregnancy to 24 weeks from 20
की ऊपरी सीमा को 20 स ाह से बढ़ाकर 24 स ाह कर दे ता है
weeks, for certain categories of women. स ाह, मिहलाओं की कुछ ेिणयों के िलए।
• S2: The opinion of a registered medical practitioner, eligible as • S2: 20 स ाह तक की अलग-अलग गभाव था की आयु म गभपात के
per the rules is required for the termination of pregnancy at
different gestation ages up to 20 weeks. Opinion of two िलए िनयमानुसार पा पं जीकृत िचिक क की राय आव क है । बीस
registered medical practitioners is required for termination स ाह से अिधक के चौबीस स ाह तक के गभ को समा करने के िलए
of pregnancy beyond twenty weeks till twenty-four weeks of दो पं जीकृत िचिक कों की राय आव क है
gestation period. प रयोजना पू री होने की अविध।
• S3: The opinion for medical termination of pregnancy
beyond twenty-four weeks gestation period will be given by a • S3: चौबीस स ाह से अिधक की गभ अविध के बाद गभपात की राय
medical board duly constituted by the respective state संबंिधत रा सरकार या क शािसत दे श शासन ारा अनुमोिदत
government or UT administration at approved facilities. Two सुिवधाओं पर िविधवत गिठत मेिडकल बोड ारा दी जाएगी। दो
registered medical practitioners eligible as per the MTP rules
एमटीपी िनयमों के अनुसार पा पं जीकृत िचिक क मेिडकल बोड के
will perform the termination of pregnancy based on the decision
of the medical board. िनणय के आधार पर गभाव था का समापन करगे।
• The rules stipulate that the medical board will have the power • िनयम िनधा रत करते ह िक मेिडकल बोड के पास केवल उिचत िवचार
to allow or deny termination of pregnancy beyond twenty-four करने और यह सुिनि त करने के बाद िक गभाव था की उ म मिहला के
weeks of gestation period only after due consideration and
िलए ि या सुरि त होगी और चाहे ू ण हो या नहीं, गभधारण की अविध
ensuring that the procedure would be safe for the woman at that
gestation age and whether the foetal malformation has substantial के चौबीस स ाह से अिधक गभाव था को समा करने की अनुमित दे ने
risk of it being incompatible with life or if the child is born it या अ ीकार करने की श होगी। कु पता से इसके जीवन के साथ
may suffer from serious physical or mental abnormalities. असंगत होने का पया जो खम होता है या यिद ब ा पै दा होता है तो वह

43. Consider the following statements with reference to Narcotic गंभीर शारी रक या मानिसक असामा ताओं से पीिड़त हो सकता है ।
Drugs and Psychotropic Substances Act, 1985:
1. Under the NDPS Act, it is illegal for a person to 43. ापक औषिध और मन: भावी पदाथ अिधिनयम, 1985 के सं दभ म
produce/manufacture/cultivate, possess, sell, purchase, transport, िन िल खत कथनों पर िवचार कर:
store, and/or consume any narcotic drug or psychotropic 1. एनडीपीएस अिधिनयम के तहत, िकसी के िलए िकसी भी मादक
substance. दवा या मन: भावी पदाथ का उ ादन/िनमाण/खेती, अिधकार, िब ी,
2. Under one of the provisions of the act, the State Governments खरीद, प रवहन, भं डारण और/या उपभोग करना अवैध है ।
are empowered to regulate the cultivation, production, 2. अिधिनयम के ावधानों म से एक के तहत, रा सरकारों को खेती,
manufacture, import, export, sale, consumption, use of narcotic उ ादन, िनमाण, आयात, िनयात, िब ी, खपत, मादक दवाओं और
drugs and psychotropic substances.
मन: भावी पदाथ के उपयोग को िविनयिमत करने का अिधकार है ।
Which of the given above statements is/are correct?
ऊपर िदए गए कथनों म से कौन-सा/से सही है /ह?
(a) 1 only
(ए) केवल 1
(b) 2 only
(बी) केवल 2
(c) Both 1 and 2
(सी) 1 और 2 दोनों
(d) Neither 1 nor 2
(डी) न तो 1 और न ही 2
Ans: (a)
Explanation: Narcotic Drugs and Psychotropic Substances उ र: (ए)
Act, 1985: ा ा: नारकोिटक ड एं ड साइकोटोिपक सब स ए , 1985:
• The Act is designed to fulfill India’s treaty obligations under • अिधिनयम को नारकोिटक ड पर एकल क शन, साइकोटोिपक
the Single Convention on Narcotic Drugs, Convention on पदाथ पर क शन, और नारकोिटक ड और अवैध त री के खलाफ
Psychotropic Substances, and United Nations Convention संयु रा क शन के तहत भारत की संिध के दािय ों को पू रा करने के
Against Illicit Traffic in Narcotic Drugs and िलए िडज़ाइन िकया गया है ।
Psychotropic Substances. साइकोटोिपक पदाथ।
• The Act 1985 sets out the statutory framework for drug law • अिधिनयम 1985 भारत म डग कानून वतन के िलए वैधािनक ढां चा
enforcement in India. िनधा रत करता है ।
• Under the Act, the cultivation, production, manufacture,
• अिधिनयम के तहत, िचिक ा या वै ािनक उ े ों को छोड़कर, मादक
possession, sale, purchase, transportation, warehousing,
consumption, inter-State movement, transshipment and import दवाओं और मनः भावी पदाथ की खेती, उ ादन, िनमाण, क ा, िब ी,
and export of narcotic drugs and psychotropic substances is खरीद, प रवहन, भं डारण, खपत, अंतर-रा आवाजाही, टां सिशपमट और
prohibited, except for medical or scientific purposes and in आयात और िनयात ितबंिधत है और म
accordance with the terms and conditions of any license, permit सरकार ारा िदए गए िकसी भी लाइसस, परिमट या ािधकरण के िनयमों
or authorization given by the Government. और शत के अनुसार।
• The Central Government is empowered to regulate the • क सरकार को ापक औषिधयों और मन: भावी पदाथ की खेती,
cultivation, production, manufacture, import, export, sale, उ ादन, िनमाण, आयात, िनयात, िब ी, खपत, उपयोग आिद को
consumption, use etc of narcotic drugs and psychotropic िविनयिमत करने का अिधकार है ।
substances. • रा सरकारों को अफीम, पो ा पु आल, औषधीय अफीम के िनमाण
• State Governments are empowered to permit and regulate
और भां ग की खेती के क े और अंतरा ीय आवाजाही की अनुमित दे ने
possession and interState movement of opium, poppy straw, the
manufacture of medicinal opium and the cultivation of cannabis और िविनयिमत करने का अिधकार है हशीश को छोड़कर।
excluding hashish. • नारकोिट ड और साइकोटोिपक पदाथ के िनमाण म इसके
• The Central Government is empowered to declare any संभािवत उपयोग के आकलन के आधार पर क सरकार को िकसी भी
substance, based on an assessment of its likely use in the पदाथ को िनयंि त पदाथ के प म घोिषत करने का अिधकार है ।
manufacture of narcotics drugs and psychotropic substances as a • क सरकार और रा सरकार दोनों को अिधिनयम के योजनों के िलए
controlled substance.
अिधका रयों को िनयु करने का अिधकार है ।
• Both the Central Government and State Governments are
empowered to appoint officers for the purposes of the Act.
44. इं िडया इनोवेशन इं डे 2022 िन िल खत म से िकसके ारा जारी
44. India Innovation Index 2022 is released by which of the िकया जाता है ?
following? (ए) नेशनल इनोवेशन फाउं डे शन
(a) National Innovation Foundation (b) नीित आयोग
(b) NITI Aayog (सी) भारतीय रजव बक
(c) Reserve Bank of India (डी) िश ा मं ालय
(d) Ministry of Education उ र: (बी)
Ans: (b) ा ा:
Explanation: • नीित आयोग का भारत नवाचार सूचकां क, 2022
• NITI Aayog’s India Innovation Index, 2022 • सूचकां क उप-रा ीय र पर नवाचार मताओं और पा र थितक तं
• The index determines innovation capacities and ecosystems at को िनधा रत करता है ।
the sub-national level. • मु प रणाम:
• Key outcomes: 0 कनाटक रिकंग म सबसे ऊपर है , इसके बाद तेलंगाना, ह रयाणा,
o Karnataka topped the ranking, followed by Telangana,
महारा और तिमलनाडु ह।
Haryana, Maharashtra and Tamil Nadu.
o मिणपु र ने पू व र और पहाड़ी रा ों की ेणी म बढ़त हािसल की
o Manipur secured the lead in the Northeast and Hill States
category क शािसत दे शों और शहर रा ों की ेणी म चंडीगढ़ शीष दशनकता
o Chandigarh was the top performer in the Union Territories and रहा।
the City States category. 0 िनचला रक: छ ीसगढ़, ओिडशा, िबहार और गुजरात सूचकां क म सबसे
o Bottom rank: Chhattisgarh, Odisha, Bihar and Gujarat were at नीचे थे।
the bottom of the index.
45. भारत के िविध आयोग के बारे म िन िल खत कथनों पर िवचार कर:
45. Consider the following statements about the law commission 1. यह भारत के सव ायालय के सलाहकार िनकाय के प म काय
of India: करता है ।
1. It works as an advisory body to the Supreme Court of India. 2. यह एक िनि त कायकाल के िलए थािपत िकया जाता है ।
2. It is established for a fixed tenure. 3. इसम क सरकार ारा िनयु कानूनी िवशेष शािमल ह।
3. It comprises legal experts appointed by the Central ऊपर िदए गए कथनों म से कौन सा/से सही है /ह?
Government.
(ए) केवल 1 और 2
Which of the statements given above is/are correct?
(बी) केवल 1 और 3
(a) 1 and 2 only
(सी) केवल 2 और 3
(b) 1 and 3 only
(डी) 1, 2 और 3
(c) 2 and 3 only
उ र: (सी)
(d) 1, 2 and 3
ा ा:
Ans: (c)
Explanation: • S1 और S2: भारत का िविध आयोग एक गैर-सां िविधक िनकाय है
• S1&S2: The Law Commission of India is a non-statutory body िजसका गठन भारत सरकार ारा हर तीन साल म िकया जाता है ।
constituted by the Indian Government every three years. • S3: एक िविध आयोग म समाज म ाय को बढ़ावा दे ने के िलए क
• S3: A law commission comprises legal experts appointed by the सरकार ारा िनयु कानूनी िवशेष शािमल होते ह।
Central Government to promote justice in society.
46. भारत म ीकर के संबंध म िन िल खत कथनों पर िवचार कर
46. Consider the following statements regarding Speaker in India 1. उसे कायकाल की सुर ा दान की जाती है ।
1. He is provided with a security of tenure. 2. उसके वेतन और भ े संसद ारा िनधा रत िकए जाते ह।
2. His salaries and allowances are fixed by Parliament. 3. एक परं परा है िक अ को अपनी पाट से इ ीफा दे ना पड़ता है और
3. There is a convention that the Speaker has to resign from his राजनीितक प से तट थ रहना पड़ता है ।
party and remain politically neutral. ऊपर िदए गए कथनों म से कौन से सही ह?
Which of the statements given above are correct?
(ए) केवल 1 और 2
(a) 1 and 2 only (बी) केवल 2 और 3
(b) 2 and 3 only (सी) केवल 1 और 3
(c) 1 and 3 only (डी) 1, 2 और 3
(d) 1, 2 and 3 उ र: (ए)
Ans: (a)
• तं ता और िन ता: चूंिक अ का पद अ िधक ित ा, थित
• Independence and Impartiality: As the office of the Speaker
is vested with great prestige, position and authority, और अिधकार से यु होता है ,
independence and impartiality become its sine qua non. The तं ता और िन ता इसकी अिनवाय यो ता बन जाती है । िन िल खत
following provisions ensure the independence and impartiality of ावधान अ के कायालय की तं ता और िन ता सुिनि त करते ह:
the office of the Speaker: • उसे कायकाल की सुर ा दान की जाती है । उसे केवल लोकसभा ारा
• He is provided with a security of tenure. He can be removed िवशेष ब मत (अथात् सदन के त ालीन सभी सद ों के ब मत) ारा पा रत
only by a resolution passed by the Lok Sabha by a special संक ारा हटाया जा सकता है , न िक सामा ब मत (अथात् सदन के
majority (ie, a majority of all the then members of the House)
सद ों के ब मत) ारा सदन म उप थत और मतदान करने वाले सद )।
and not by an ordinary majority (ie, a majority of
िन ासन के इस ाव पर तभी िवचार और चचा हो सकती है जब इसे कम
the members present and voting in the House). This motion of
से कम 50 सद ों का समथन ा हो।
removal can be considered and discussed only when it has the
support of at least 50 members. • उनके वेतन और भ े संसद ारा तय िकए जाते ह। वे भारत की संिचत
• His salaries and allowances are fixed by Parliament. They िनिध पर भा रत होते ह और इस कार संसद के वािषक मतदान के अधीन
are charged on the Consolidated Fund of India and thus are not नहीं होते ह।
subject to the annual vote of Parliament. 0 उनके काय और आचरण पर लोकसभा म एक ठोस ाव के अलावा
o His work and conduct cannot be discussed and criticized in the चचा और आलोचना नहीं की जा सकती है ।
Lok Sabha except on a substantive motion. o ि या को िविनयिमत करने या ापार करने या संचािलत करने की
o His powers of regulating procedure or conducting business or उनकी श यां सदन म व था बनाए रखना िकसी ायालय के
maintaining order in the House are not subject to the jurisdiction े ािधकार के अधीन नहीं है ।
of any Court. o वह पहली बार म मतदान नहीं कर सकता है । मत बराबर होने की थित
o He cannot vote in the first instance. He can only exercise a
म ही वह िनणायक मत का योग कर सकता है । यह अ की थित को
casting vote in the event of a tie. This makes the position of
Speaker impartial. िन बनाता है ।
o He is given a very high position in the order of precedence. He o उसे वरीयता के म म ब त ऊँचा थान िदया जाता है । उ भारत के
is placed at seventh rank, along with the Chief Justice of India. मु ायाधीश के साथ सातव थान पर रखा गया है । इसका मतलब है ,
This means, he has a higher rank than all cabinet ministers, धान मं ी या उप धान मं ी को छोड़कर, उनके पास सभी कैिबनेट
except the Prime Minister or Deputy Prime Minister. मंि यों की तुलना म उ रक है ।
• In Britain, the Speaker is strictly a nonparty man. There is • ि टे न म, अ स ी से एक गैर-दलीय होता है । एक परं परा है
a convention that the Speaker has to resign from his party िक अ को अपनी पाट से इ ीफा दे ना पड़ता है और राजनीितक प
and remain politically neutral. से तट थ रहना पड़ता है ।
• This healthy convention is not fully established in India where • यह थ प रपाटी भारत म पूरी तरह से थािपत नहीं है जहां अ
the Speaker does not resign from the membership of his party on
उ पद के िलए चुने जाने पर अपनी पाट की सद ता से इ ीफा नहीं
his election to the exalted office.
दे ता है ।
47. Consider the following statements:
1. Like the Speaker in Lok Sabha, the Chairman of Rajya Sabha 47. िन िल खत कथनों पर िवचार कीिजएः
is empowered to “direct any Member whose conduct is in his 1. लोक सभा के अ की तरह, रा सभा के सभापित को "िकसी भी
opinion grossly disorderly to withdraw immediately” from the सद को, िजसका आचरण उसकी राय म घोर अ व थत आचरण है ,
House. तुरंत सभा से बाहर जाने का िनदश दे ने" का अिधकार है ।
2. Unlike the Speaker, the Rajya Sabha Chairman does not have 2. अ के िवपरीत, रा सभा के सभापित के पास िकसी सद को
the power to suspend a Member. िनलंिबत करने की श नहीं होती है ।
Which of the statements given above is/are correct? ऊपर िदए गए कथनों म से कौन सा/से सही है /ह?
(a) 1 only (ए) केवल 1
(b) 2 only (बी) केवल 2
(c) Both 1 and 2 (सी) 1 और 2 दोनों
(d) Neither 1 nor 2 (डी) न तो 1 और न ही 2
Ans: (c) उ र: (सी)
Explanation: ा ा:
• Like the Speaker in Lok Sabha, the Chairman of Rajya Sabha is • लोक सभा के अ की तरह, रा सभा के सभापित को इसकी िनयम
empowered under Rule Number 255 of its Rule Book to “direct
पु का के िनयम सं ा 255 के तहत "िकसी भी सद को, िजसका
any Member whose conduct is in his opinion grossly disorderly
आचरण उनकी राय म घोर उ ृं खल है , त ाल सदन से बाहर जाने का
to withdraw immediately” from the House.
• Unlike the Speaker, however, the Rajya Sabha Chairman does िनदश" दे ने का अिधकार है ।
not have the power to suspend a Member. •अ के िवपरीत, हालां िक, रा सभा के सभापित के पास िकसी सद
को िनलंिबत करने की श नहीं है ।
48. Consider the following statements:
1. Under Article 82 of the Constitution of India, the Parliament 48. िन िल खत कथनों पर िवचार कीिजएः
enacts a Delimitation Act after every Census. 1. भारत के संिवधान के अनु े द 82 के तहत, संसद ेक जनगणना के
2. Under Article 170 of the Constitution of India, States also get बाद एक प रसीमन अिधिनयम बनाती है ।
divided into territorial constituencies as per the Delimitation Act 2. भारत के संिवधान के अनु े द 170 के तहत, रा ों को हर जनगणना
after every Census. के बाद प रसीमन अिधिनयम के अनुसार े ीय िनवाचन े ों म िवभािजत
Which of the given above statements is/are correct? िकया जाता है ।
(a) 1 only ऊपर िदए गए कथनों म से कौन-सा/से सही है /ह?
(b) 2 only (ए) केवल 1
(c) Both 1 and 2 (बी) केवल 2
(d) Neither 1 nor 2 (सी) 1 और 2 दोनों
Ans: (c) (डी) न तो 1 और न ही 2
Explanation: उ र: (सी)
• What is Delimitation:- Delimitation literally means the ा ा:
process of fixing limits or boundaries of territorial constituencies • प रसीमन ा है :- प रसीमन का शा क अथ है एक िवधायी िनकाय
in a state that has a legislative body.
वाले रा म े ीय िनवाचन े ों की सीमा या सीमाओं को तय करने की
• Delimitation commission orders have the force of law and
ि या।
they cannot be challenged before any court.
• Composition of the Commission: According to the • प रसीमन आयोग के आदे शों म कानून का बल है और उ िकसी भी
Delimitation Commission Act, 2002, the Delimitation अदालत के सम चुनौती नहीं दी जा सकती है ।
Commission will have three members: a serving or retired • आयोग की संरचना: प रसीमन आयोग अिधिनयम, 2002 के अनुसार,
judge of the Supreme Court as the chairperson, and the Chief प रसीमन आयोग म तीन सद होंगे: अ के प म सव ायालय
Election Commissioner or Election Commissioner nominated by का एक सेवारत या सेवािनवृ ायाधीश, और मु चुनाव आयु या
the CEC and the State Election Commissioner as ex-officio सीईसी ारा नािमत चुनाव आयु और पदे न सद ों के प म रा
members. िनवाचन आयु ।
• Constitutional Provisions: • संवैधािनक ावधान:
o Under Article 82, the Parliament enacts a Delimitation Act
o अनु े द 82 के तहत, संसद हर जनगणना के बाद एक प रसीमन
after every Census.
o Under Article 170, States also get divided into territorial अिधिनयम बनाती है ।
constituencies as per the Delimitation Act after every Census. 0 अनु े द 170 के तहत, रा ों को ेक जनगणना के बाद प रसीमन
अिधिनयम के अनुसार े ीय िनवाचन े ों म िवभािजत िकया जाता है ।
49. Along with the budget, the Finance Minister also places
other documents before the parliament which include “The 49. बजट के साथ, िव मं ी संसद के सम अ द ावेज भी रखता है
Macro Economic Framework Statement” The aforesaid िजसम "मै ो इकोनॉिमक े मवक े टमट" शािमल है । उपरो द ावेज़
document is presented because this mandated by ु त िकया गया है ोंिक यह अिनवाय है
(a) Long standing parliamentary convention (ए) लंबे समय से चली आ रही संसदीय परं परा
(b) Article 112 and article 110(1) of the constitution of India (बी) भारत के संिवधान के अनु े द 112 और अनु े द 110 (1)।
(c) Article 114 of the Constitution of India (सी) भारत के संिवधान के अनु े द 114
(d) Provisions of the Fiscal Responsibility and Budget (d) राजकोषीय उ रदािय और बजट बंधन अिधिनयम, 2003 के
Management act, 2003 ावधान
Ans: (d) उ र: (डी)
Explanation: ा ा:
• The Macro-economic Framework Statement is a statement
• मै ो-इकोनॉिमक े मवक े टमट एक े टमट है िजसे ु त िकया
presented to the
जाता है
Parliament at the time of Union Budget under Section 3(5) of the
Fiscal Responsibility and Budget Management Act, 2003.] राजकोषीय उ रदािय और बजट बंधन अिधिनयम, 2003 की धारा
3(5) के तहत क ीय बजट के समय संसद।]
50. With reference to Foreign Contribution Regulation
(Amendment), Act 2020, which of the following statements 50. िवदे शी अंशदान िविनयमन (संशोधन), अिधिनयम 2020 के सं दभ म,
is/are correct? िन िल खत म से कौन सा/से कथन सही है /ह?
1. Under the Act, foreign contribution cannot be transferred to 1. अिधिनयम के तहत, िवदे शी अंशदान िकसी अ को तब तक
any other person unless such person is also registered for that ह ां त रत नहीं िकया जा सकता जब तक िक वह भी उस उ े
purpose. के िलए पं जीकृत न हो।
2. The act states that foreign contributions must be received only 2. अिधिनयम म कहा गया है िक भारतीय े ट बक, नई िद ी शाखा म
in an FCRA account opened in the State Bank of India, New खोले गए एफसीआरए खाते म ही िवदे शी योगदान ा होना चािहए।
Delhi Branch. 3. अिधिनयम ने िवदे शी अंशदान (िविनयमन) अिधिनयम, 2010 की तुलना
3. The act increased administrative expenses through foreign म एक संगठन ारा िवदे शी िनिधयों के मा म से शासिनक य म वृ
funds by an organization compared to Foreign Contribution
की।
(Regulation) Act, 2010.
नीचे िदए गए कूट का योग कर सही उ र चुिनए:
Select the correct answer using the code below:
(ए) केवल 1 और 2
(a) 1 and 2 only
(बी) केवल 2 और 3
(b) 2 and 3 only
(सी) केवल 1 और 3
(c) 1 and 3 only
(डी) 1, 2 और 3
(d) 1, 2 and 3
Ans: (a) उ र: (ए)
Explanation; Foreign Contribution Regulation (Amendment), ा ा; िवदे शी अंशदान िविनयमन (संशोधन), अिधिनयम 2020:
Act 2020: • अिधिनयम के तहत, िवदे शी अंशदान को थानां त रत नहीं िकया जा
• Under the Act, foreign contribution cannot be transferred सकता है
to any other person unless such person is also registered for िकसी अ को जब तक िक ऐसा भी उस उ े के िलए
that purpose. पं जीकृत न हो।
• The amendment also forbids sub-granting by NGOs to smaller • संशोधन जमीनी र पर काम करने वाले छोटे एनजीओ को एनजीओ
NGOs who work at the grassroots. ारा सब- ां ट दे ने पर भी रोक लगाता है ।
• The act states that foreign contributions must be received
• अिधिनयम म कहा गया है िक भारतीय े ट बक, नई िद ी शाखा म
only in an FCRA account opened in the State Bank of India,
New Delhi Branch. No funds other than the foreign contribution खोले गए एफसीआरए खाते म ही िवदे शी अंशदान ा िकया जाना
should be received or deposited in this account. चािहए। इस खाते म िवदे शी अंशदान के अलावा कोई भी धनरािश ा या
• Aadhar usage: The act makes it compulsory for all trustees to जमा नहीं की जानी चािहए।
register their Aadhaar card with the FCRA account. • आधार का उपयोग: अिधिनयम सभी ट यों के िलए एफसीआरए खाते
• The Act also makes Aadhaar a mandatory identification के साथ अपने आधार काड को पंजीकृत करना अिनवाय बनाता है ।
document. It is for all the office bearers, directors, and other key • अिधिनयम आधार को एक अिनवाय पहचान द ावेज भी बनाता है । यह
functionaries of an NGO. एक एनजीओ के सभी पदािधका रयों, िनदे शकों और अ मुख
• Restriction in utilisation of foreign contribution: The act gives पदािधका रयों के िलए है ।
government powers to stop utilization of foreign funds by an • िवदे शी अंशदान के उपयोग पर ितबंध: अिधिनयम एक "सारां श जां च"
organization through a “summary enquiry”.
के मा म से एक संगठन ारा िवदे शी धन के उपयोग को रोकने के िलए
• Reduction in use of foreign contribution for administrative
सरकारी श यां दे ता है ।
purposes: The act decreases administrative expenses through
foreign funds by an organization to 20% from 50% earlier. • शासिनक उ े ों के िलए िवदे शी योगदान के उपयोग म कमी:
• Surrender of certificate: The act allows the central government अिधिनयम एक संगठन ारा िवदे शी धन के मा म से शासिनक य को
to permit a person to surrender their registration certificate. 50% से घटाकर 20% कर दे ता है ।
• सरडर ऑफ सिटिफकेट: ए क सरकार को अनुमित दे ता है िक वह
51. Consider the following statements about Comptroller and िकसी को अपना रिज े शन सिटिफकेट सरडर करने की अनुमित
Auditor General of India (CAG): दे सकता है ।
1. The CAG is mentioned in the Constitution of India.
2. He is the head of the Indian Audit and Accounts Department. 51. भारत के िनयं क और महालेखा परी क (CAG) के बारे म
3. His duty is to uphold the Constitution of India and the laws of िन िल खत कथनों पर िवचार कर:
Parliament in the field of financial administration. 1. CAG का उ ेख भारत के संिवधान म िकया गया है ।
Which of the statements given above is/are correct? 2. वह भारतीय लेखापरी ा और लेखा िवभाग के मुख ह।
(a) 1 and 2 only 3. उनका कत िव ीय शासन के े म भारत के संिवधान और संसद
(b) 2 and 3 only के कानूनों को बनाए रखना है ।
(c) 1 and 3 only ऊपर िदए गए कथनों म से कौन सा/से सही है /ह?
(d) 1, 2 and 3 (ए) केवल 1 और 2
Ans: (d)
(बी) केवल 2 और 3
Explanation:
(सी) केवल 1 और 3
• The Constitution of India provides for an independent office of
the Comptroller and Auditor General of India (CAG) in chapter (डी) 1, 2 और 3
V under Part V. उ र: (डी)
• The CAG is mentioned in the Constitution of India under ा ा:
Article 148 – 151. • भारत का संिवधान भाग V के तहत अ ाय V म भारत के िनयं क और
• He is the head of the Indian Audit and Accounts Department. महालेखा परी क (CAG) के एक तं कायालय का ावधान करता है ।
• He is the guardian of the public purse and controls the entire • CAG का उ ेख भारत के संिवधान म अनु े द 148 - 151 के तहत
financial system of the country at both the levels- the centre and िकया गया है ।
state. • वह भारतीय लेखापरी ा और लेखा िवभाग के मुख ह।
• His duty is to uphold the Constitution of India and the laws of
• वह सावजिनक धन का संर क होता है और क और रा दोनों रों
Parliament in the field of financial administration.
पर दे श की संपूण िव ीय णाली को िनयंि त करता है ।
• उनका कत िव ीय शासन के े म भारत के संिवधान और संसद के
52. 256. With reference to ‘Seekho aur Kamao’ (Learn and
Earn) scheme, consider the following statements: कानूनों को बनाए रखना है ।
1. It is a Skill development scheme for the youth of the 19 – 30
years age group. 52. 256. 'सीखो और कमाओ' (सीखो और कमाओ) योजना के संदभ म
2. The scheme implemented by the Ministry of Minority Affairs. िन िल खत कथनों पर िवचार कीिजए:
Which of the statements given above is/are correct? 1. यह 19-30 वष आयु वग के युवाओं के िलए कौशल िवकास योजना है ।
(a) 1 only 2. अ सं क काय मं ालय ारा लागू की गई योजना।
(b) 2 only ऊपर िदए गए कथनों म से कौन सा/से सही है /ह?
(c) Both 1 and 2 (ए) केवल 1
(d) Neither 1 nor 2 (बी) केवल 2
Ans: (b) (सी) 1 और 2 दोनों
Explanation: About the Scheme (डी) न तो 1 और न ही 2
• Nodal ministry: Central Sector Scheme under the Ministry उ र: (बी)
of Minority Affairs (since 2013-14)
• Aim: Upgrading the skills of minority youth (14-35 years ीकरण: योजना के बारे म
age group) and ensure 75% placements, out of which 50% • नोडल मं ालय: अ सं क मामलों के मं ालय के तहत क ीय े की
should be in the organized sector. योजना (2013-14 से)
• उ े : अ सं क यु वाओं (14-35 वष आयु वग) के कौशल का
53. ‘Naya Savera Scheme’ is related/aims to
उ यन और 75% ेसमट सुिनि त करना, िजसम से 50% संगिठत े म
(a) Leadership development of women belonging to minority
होना चािहए।
communities
(b) Upgrading the Skills and Training in Traditional Arts/Crafts
for Development 53. 'नया सवेरा योजना' संबंिधत/उ े से संबंिधत है
(c) interest subsidy on educational loans for overseas higher (ए) अ सं क समुदायों से संबंिधत मिहलाओं का नेतृ िवकास
studies (बी) िवकास के िलए पारं प रक कलाओं/िश ों म कौशल और िश ण
(d) provide free coaching to minority students for the various का उ यन
competitive exam (सी) िवदे शी उ अ यन के िलए शैि क ऋण पर ाज स डी
Ans: (d) (डी) अ सं क छा ों को िविभ ितयोगी परी ाओं के िलए मु
Explanation: कोिचंग दान करना
• Naya Savera Scheme (to provide free coaching to minority उ र: (डी)
students for the various competitive exam) ा ा:
• Padho Pardesh Scheme (interest subsidy on educational loans • नया सवेरा योजना (िविभ ितयोगी परी ाओं के िलए अ सं क छा ों
for overseas higher studies) को मु कोिचंग दान करने के िलए)
• Nai Udaan Scheme (supports students clearing Prelims of
• पढ़ो परदे श योजना (िवदे श म उ अ यन के िलए शैि क ऋण पर
UPSC, SPSC or other exams)
ाज स डी)
• Nai Roshni Scheme ( Leadership development of women
belonging to minority communities) • नई उड़ान योजना (यूपीएससी, एसपीएससी या अ परी ाओं की
• USTTAD (Upgrading the Skills and Training in Traditional ारं िभक परी ा उ ीण करने वाले छा ों को सहायता)
Arts/Crafts for Development) • नई रोशनी योजना (अ सं क समुदायों से संबंिधत मिहलाओं का
• Nai Manzil Scheme (for formal school education & skilling नेतृ िवकास)
of school dropouts) • उ ाद (िवकास के िलए पारं प रक कला/िश म कौशल और िश ण
• Hamari Dharohar (to preserve the rich heritage of minority का उ यन)
communities of India) • नई मंिज़ल योजना (औपचा रक ू ली िश ा और ू ल छोड़ने वालों के
कौशल िवकास के िलए)
54. With reference to United Nations Human Rights Council • हमारी धरोहर (भारत के अ सं क समुदायों की समृ िवरासत को
(UNHRC), consider the
संरि त करने के िलए)
following statements:
1. The United Nations Human Rights Council (UNHRC) is a
54. संयु रा मानवािधकार प रषद (यूएनएचआरसी) के संदभ म िवचार
United Nations body whose mission is to promote and protect
human rights around the world. कर
2. The Council has 47 members elected for staggered three-year िन िल खत बयान:
terms on a regional group basis. 1. संयु रा मानवािधकार प रषद (UNHRC) एक संयु रा िनकाय है
3. The headquarters of the Council is in Geneva, Switzerland. िजसका िमशन दु िनया भर म मानवािधकारों को बढ़ावा दे ना और उनकी
Which of the statements given above is/are correct? र ा करना है ।
(a) 1 and 2 only 2. प रषद म 47 सद े ीय समूह के आधार पर तीन साल की अविध के
(b) 2 and 3 only िलए चुने गए ह।
(c) 1 and 3 only 3. प रषद का मु ालय िजनेवा, ट् जरलड म है ।
(d) 1, 2 and 3 ऊपर िदए गए कथनों म से कौन सा/से सही है /ह?
Ans: (d) (ए) केवल 1 और 2
Explanation: (बी) केवल 2 और 3
• The United Nations Human Rights Council (UNHRC) is a (सी) केवल 1 और 3
United Nations body whose mission is to promote and protect (डी) 1, 2 और 3
human rights around the world. The Council has 47 members उ र: (डी)
elected for staggered three-year terms on a regional group
ा ा:
basis. The headquarters of the Council is in Geneva,
• संयु रा मानवािधकार प रषद (यूएनएचआरसी) एक संयु रा िनकाय है
Switzerland.
िजसका िमशन दु िनया भर म मानवािधकारों को बढ़ावा दे ना और उनकी र ा
• Functions:
करना है । प रषद म े ीय समू ह के आधार पर कंिपत तीन साल की शत के
• The UNHRC passes non-binding resolutions on human rights
issues through a periodic review of all 193 UN member states िलए चुने गए 47 सद ह। प रषद का मु ालय िजने वा, ट् जरलड म है ।
called the Universal Periodic Review (UPR). • काय:
• It oversees expert investigation of violations in specific • यूएनएचआरसी सभी 193 संयु रा सद रा ों की आविधक समी ा के
countries (Special Procedures). मा म से मानवािधकारों के मु ों पर गैर-बा कारी संक पा रत करता है
िजसे यूिनवसल पी रयोिडक र ू (यूपीआर) कहा जाता है ।
• यह िविश दे शों (िवशेष ि याओं) म उ ंघनों की िवशेष जां च की
दे खरे ख करता है ।

You might also like